[obm-l] Re: [obm-l] Ângulos de um triângulo

2020-12-05 Por tôpico Julio César Saldaña Pumarica
É verdade, 30 graus é o DAB, más a pergunta era DAC

o  DAC=18


On Fri, Dec 4, 2020, 19:23 Julio César Saldaña Pumarica <
saldana...@pucp.edu.pe> wrote:

> Tenho uma solução com traços auxiliares. Resposta: 30°
>
> Tem como passar uma foto nesta lista?
>
> On Mon, Nov 30, 2020, 19:42 Professor Vanderlei Nemitz <
> vanderma...@gmail.com> wrote:
>
>> Boa noite!
>> Alguém conhece uma saída para o seguinte problema?
>> Muito obrigado!
>>
>> *Num triângulo isósceles ABC, AB = AC.*
>> *Seja D um ponto interno tal que os ângulos DBC, DCB, DBA e DCA medem,
>> respectivamente, 12°, 18°, 54° e 48°. *
>> *Determine a medida do ângulo DAC.*
>>
>>
>> <https://www.avast.com/sig-email?utm_medium=email_source=link_campaign=sig-email_content=webmail>
>>  Livre
>> de vírus. www.avast.com
>> <https://www.avast.com/sig-email?utm_medium=email_source=link_campaign=sig-email_content=webmail>.
>>
>> <#m_-1377089701738023066_m_6233189016624438778_m_1892860381202400781_DAB4FAD8-2DD7-40BB-A1B8-4E2AA1F9FDF2>
>>
>


[obm-l] Re: [obm-l] Ângulos de um triângulo

2020-12-04 Por tôpico Julio César Saldaña Pumarica
Tenho uma solução com traços auxiliares. Resposta: 30°

Tem como passar uma foto nesta lista?

On Mon, Nov 30, 2020, 19:42 Professor Vanderlei Nemitz <
vanderma...@gmail.com> wrote:

> Boa noite!
> Alguém conhece uma saída para o seguinte problema?
> Muito obrigado!
>
> *Num triângulo isósceles ABC, AB = AC.*
> *Seja D um ponto interno tal que os ângulos DBC, DCB, DBA e DCA medem,
> respectivamente, 12°, 18°, 54° e 48°. *
> *Determine a medida do ângulo DAC.*
>
>
> 
>  Livre
> de vírus. www.avast.com
> .
> <#m_1892860381202400781_DAB4FAD8-2DD7-40BB-A1B8-4E2AA1F9FDF2>
>


Re: [obm-l]

2019-11-29 Por tôpico Julio César Saldaña Pumarica
On Thu, Nov 28, 2019, 14:47 Julio César Saldaña Pumarica <
saldana...@pucp.edu.pe> wrote:

> Pensei numa solução baseada no problema 2 da 1era olimpiada iberoamericana
> de matemática. Mas me parece que vai precisar de muita fibra muscular
> algébrica.
>
> Numa solução daquele problema, desenhavam-se triângulos exteriores sobre
> os lados do triângulo equilátero. Um teria lados L-a-b, outro L-b-c e outro
> L-a-c (eles são congruentes aos triângulos internos) . Bom, o resultado era
> um hexágono, e a área daquele hexágono podia calcular se de duas maneiras:
>
> 1. Somando o triângulo equilátero L-L-L aos três acima mencionados, o que
> resulta em 2 vezes área de L-L-L:  2 x L^2 x sqrt(3)/4
>
> 2. Somando 6 triângulos: 3 deles são equiláteros a-a-a, b-b-b, c-c-c, e os
> outros 3 son iguais e de lados a-b-c (fórmula de heron, onde aparecem a^4,
> b^4, c^4)
>
> Igualando as duas áreas e com algum esforço algébrico deve sair aquela
> relação.
>
> Quem sabe tem algum método trigonométrico mais simples
>
> Atenciosamente
>
> Julio
>
>
>
> El mar., 26 nov. 2019 a las 20:34, gilberto azevedo ()
> escribió:
>
>> Pesquisando achei uma relação muito interessante, mas não achei nenhuma
>> demonstração dela na web.
>> Pra quem se interessar Seja um ponto P no interior de um triângulo
>> equilátero de lado l, e a,b,c a distância desse ponto aos vértices do
>> triângulo. Provar que :
>> 3( a⁴ + b⁴ + c⁴ + l⁴) = ( a² + b² + c² + l²)²
>>
>> --
>> Esta mensagem foi verificada pelo sistema de antivírus e
>> acredita-se estar livre de perigo.
>
>

-- 
Esta mensagem foi verificada pelo sistema de antiv�rus e
 acredita-se estar livre de perigo.



Re: [obm-l]

2019-11-28 Por tôpico Julio César Saldaña Pumarica
Pensei numa solução baseada no problema 2 da 1era olimpiada iberoamericana
de matemática. Mas me parece que vai precisar de muita fibra muscular
algébrica.

Numa solução daquele problema, desenhavam-se triângulos exteriores sobre os
lados do triângulo equilátero. Um teria lados L-a-b, outro L-b-c e outro
L-a-c (eles são congruentes aos triângulos internos) . Bom, o resultado era
um hexágono, e a área daquele hexágono podia calcular se de duas maneiras:

1. Somando o triângulo equilátero L-L-L aos três acima mencionados, o que
resulta em 2 vezes área de L-L-L:  2 x L^2 x sqrt(3)/4

2. Somando 6 triângulos: 3 deles são equiláteros a-a-a, b-b-b, c-c-c, e os
outros 3 son iguais e de lados a-b-c (fórmula de heron, onde aparecem a^4,
b^4, c^4)

Igualando as duas áreas e com algum esforço algébrico deve sair aquela
relação.

Quem sabe tem algum método trigonométrico mais simples

Atenciosamente

Julio



El mar., 26 nov. 2019 a las 20:34, gilberto azevedo ()
escribió:

> Pesquisando achei uma relação muito interessante, mas não achei nenhuma
> demonstração dela na web.
> Pra quem se interessar Seja um ponto P no interior de um triângulo
> equilátero de lado l, e a,b,c a distância desse ponto aos vértices do
> triângulo. Provar que :
> 3( a⁴ + b⁴ + c⁴ + l⁴) = ( a² + b² + c² + l²)²
>
> --
> Esta mensagem foi verificada pelo sistema de antivírus e
> acredita-se estar livre de perigo.

-- 
Esta mensagem foi verificada pelo sistema de antiv�rus e
 acredita-se estar livre de perigo.



Re: [obm-l]

2019-11-22 Por tôpico Julio César Saldaña Pumarica
mudando a variável:

x-12 = y^6

El vie., 22 nov. 2019 a las 2:40, gilberto azevedo ()
escribió:

> Como achar o mínimo de :
> x² * √(x²/(x-12)) , usando apenas desigualdades comuns ?
>
> --
> Esta mensagem foi verificada pelo sistema de antivírus e
> acredita-se estar livre de perigo.

-- 
Esta mensagem foi verificada pelo sistema de antiv�rus e
 acredita-se estar livre de perigo.



Re: [obm-l] Geometria triangulo

2019-04-05 Por tôpico Julio César Saldaña Pumarica
Trace DP perpendicular a BE com P em BC, logo BP=BD. Seja Q o ponto comum a
DP e BE
Calculando os ângulos (os que dá para calcular), obtemos ) escribió:

> Alguem temnuma construcao esperta pra essa?
>
> Num triangulo retangulo ABC , retangulo em A , o angulo ABC=20 graus, traca-se
> a bissetriz deste  angulo que toca o lado AC em E. Em seguida, traca-se a
> reta CD com D em AB tal que ACD=30, determinar o angulo CDE.
>
> Douglas Oliveira.
>
>
> --
> Esta mensagem foi verificada pelo sistema de antivírus e
> acredita-se estar livre de perigo.

-- 
Esta mensagem foi verificada pelo sistema de antiv�rus e
 acredita-se estar livre de perigo.



Re: [obm-l] Geometria plana

2019-02-25 Por tôpico Julio César Saldaña Pumarica
Na prolongação do BP ubique o ponto Q tal que AQ=AB. Chamemos AC=R, então
temos AB=AC+AQ=R.

Completando ângulos: ) escribió:

> Ola amigos, alguem ja fez essa questao abaixo?
>
> Eu fiz por trigonometria e achei 80 graus.
> Gostaria de uma ajuda para fazer por construcao.
>
> Problema:
> Num triangulo ABC isosceles , onde AB=AC, o angulo A mede 40 graus,
> traca-se BP com P em AC, e o angulo ABP mede 20 graus. Toma-se um ponto M
> em BP de modo que AP=PM, determinar o angulo PMC.
>
> Valeh pela ajuda
> Douglas Oliveira.
>
> --
> Esta mensagem foi verificada pelo sistema de antivírus e
> acredita-se estar livre de perigo.

-- 
Esta mensagem foi verificada pelo sistema de antiv�rus e
 acredita-se estar livre de perigo.



[obm-l] Re: [obm-l] Sequência de Fibonacci

2019-02-15 Por tôpico Julio César Saldaña Pumarica
Tal vez isto seja indução, mas vou compartilhar mesmo assim:

Defina:  A_m = F_2m •F_m-1 - F_2m-1•F_m   .(1)
Defina: B_m = (-1)^m x A_m   ...(2)
Calculando  B_(m+1)-B_(m-1) e com um pouco de suor obtemos B_(m+1)-B_(m-1)=B_m,
ouseja, B_m segue a regra de Fibonacci, além de mais B_1=F_1, B_2=F_2

Logo B_m = F_m (tal vez esta conclusão só se sustenta por indução)

Substituindo em (2): F_m = (-1)^m x A_m, logo A_m = (-1)^m x F_m


El mié., 13 feb. 2019 a las 21:37, Jeferson Almir ()
escribió:

> Como provar esse resultado de fibonacci que não seja por indução ??
> F_2m •F_m-1 - F_2m-1•F_m = (-1)^m•F_m
>
> --
> Esta mensagem foi verificada pelo sistema de antivírus e
> acredita-se estar livre de perigo.

-- 
Esta mensagem foi verificada pelo sistema de antiv�rus e
 acredita-se estar livre de perigo.



[obm-l] Re: [obm-l] Função Composta

2018-05-11 Por tôpico Julio César Saldaña Pumarica
com isso prova que f nao pode ser linear mas o enunciado pareces mais geral

El viernes, 11 de mayo de 2018, Rodrigo Ângelo 
escribió:

> Se f : |N -> |N, f(n) = an + m, com a e m constantes naturais, então
> teríamos
> f(f(n)) = a(an + m) + m
> f(f(n)) = (a^2)n + am + m
>
> Com f(f(n)) = n + 2005, teríamos a = 1 e m = 2005/2, absurdo, pois m deve
> ser um número natural.
>
> On Fri, May 11, 2018 at 10:51 AM Jeferson Almir 
> wrote:
>
>> Como provar que nos naturais não existe a função f ( f(n) ) = n + 2005 ???
>>
>> --
>> Esta mensagem foi verificada pelo sistema de antivírus e
>> acredita-se estar livre de perigo.
>
>
> --
> Esta mensagem foi verificada pelo sistema de antivírus e
> acredita-se estar livre de perigo.

-- 
Esta mensagem foi verificada pelo sistema de antiv�rus e
 acredita-se estar livre de perigo.



[obm-l] Re: [obm-l] Triângulo quase Russo - 12º - 18º - 42º

2018-04-04 Por tôpico Julio César Saldaña Pumarica
Ontem enviei uma solução como arquivo anexo. Era uma foto com a minha
solução. Parece que o email não chegou, poderia me confirmar?, existe
alguma restrição quanto anexos?

A resposta é 48, e fiz a solução usando apenas geometria básica.

Obrigado

Julio

2018-02-28 7:36 GMT-03:00 Jeferson Almir :

> Queria uma ajuda nesse problema de preferência por geometria sintética :)
>
> Seja um triângulo ABC isósceles de base BC sendo  A = 12º e os pontos E e
> D sobre AB e BC respectivamente tal que os ângulos ECB= 42º e DBC =18º.
> Calcule o ângulo EDB.
>
> --
> Esta mensagem foi verificada pelo sistema de antivírus e
> acredita-se estar livre de perigo.

-- 
Esta mensagem foi verificada pelo sistema de antiv�rus e
 acredita-se estar livre de perigo.



Re: [obm-l] Geometria plana

2018-04-02 Por tôpico Julio César Saldaña Pumarica
Pensei na seguinte solução usando congruência de triângulos

1. Pela condição do perímetro podemos deduzir que PQ=PB+QD

2. Estique o segemento AB até o ponto T tal que BT=QD, então os triângulos
TBC e QCD são congruentes pelo caso L.A.L.; e portanto concluimos que
CT=CQ. Notemos também que PT=PQ

3. Calculemos o ângulo TCQ. Este é igual a TCB+BCQ, mas como TCB=QCD (pela
congruência do ponto 2), então TCQ=QCD+BCQ=90

4. Finalmente notemos que os triângulos TCP e PCQ são congruentes pelo caso
L.L.L, e portanto os ângulos TCP e PCQ são iguais, e como ambos somam 90,
cada um deve ser 45

Julio

2018-04-02 16:36 GMT-03:00 Claudio Buffara :

> Tudo bem.
> Mas minha dúvida é outra: como/por que você pensou em usar a
> circunferência centrada em C e passando por B e D?
>
> Este é um dos temas que mais me interessa em matemática: de onde vêm as
> idéias não óbvias?
> Inspiração divina?
> Experiência ("já vi algo parecido antes")?
> Muita transpiração?
>
> Pois, neste problema, após chamar AP de x e AQ de y, minha primeira
> tentativa foi usar Pitágoras para determinar CP e CQ e daí usar a lei dos
> cossenos pra determinar o ângulo PCQ. Caí num emaranhado algébrico.
> Daí, me ocorreu a ideia de calcular PCQ como sendo 90 - (PCB + QCD) e, ao
> fazer isso, percebi que tan(PCB) = 1-x, etc... Com um pouco de
> trigonometria e álgebra, cheguei à resposta.
>
> Mas, a meu ver, a solução mais elegante é a que usa o círculo. Só que não
> é óbvio, a priori, que aquele círculo ajuda.
>
> Mesmo levando em conta que, pelo enunciado, o ângulo PCQ provavelmente
> deve ter medida invariante, independentemente de AP e AQ, ainda assim não é
> óbvio (pelo menos pra mim) que o círculo mata o problema. Por outro lado,
> se o ângulo PCQ tem medida invariante, então tomando o caso extremo P = B e
> Q = A (portanto, o triângulo APQ é degenerado) constatamos facilmente que
> PCQ (no caso, ACB) deve medir 45 graus.
>
> []s,
> Claudio.
>
>
>
> 2018-04-02 16:13 GMT-03:00 Douglas Oliveira de Lima <
> profdouglaso.del...@gmail.com>:
>
>> Da para fazer uma prova por absurdo.
>> Fica bom, suponha que a reta nao tangencia a circunferencia entao trace a
>> tangente e vai chegar em um absurdo.
>>
>> Abraco
>> Douglas Oliveira.
>>
>> Em seg, 2 de abr de 2018 11:14, Claudio Arconcher 
>> escreveu:
>>
>>> Bom dia caros colegas.
>>>
>>> Ponhamos ABCD o quadrado (o ponto A está no lado de baixo e à esquerda,
>>> segue-se o ponto B à direita, C e D estão no lado de acima fechando o
>>> circuito ABCD ).
>>>
>>> Ponhamos: AP=x e AQ=y, segue-se, QD=1-y e PB=1-x.
>>>
>>> Tracemos a circunferência de centro C e raio 1, ela tangencia AD em D e
>>> AB em B, agora seja M um ponto no quarto dessa circunferência interno ao
>>> quadrado ABCD e tracemos a tangente a ela por M, cortando AD em Q e AB em P
>>> ( serão, de fato os pontos esperados ), tem-se: QD=1-y = QM e PB=1 – x =
>>> PM, o perímetro do triângulo retângulo QAP é igual a 2. Reciprocamente se
>>> consideramos o triângulo AQP de perímetro 2 fixado antes o ponto M será o
>>> mesmo, todos esses triângulos são assim obtidos, com PQ tangente à
>>> circunferência em um ponto M com a propriedade descrita.
>>>
>>> Agora basta examinar as congruências dos triângulos retângulos CDQ e CMQ
>>> e, também, CBP e CMP, isso nos leva a concluir que o ângulo PCQ mede 45 º.
>>>
>>> Espero que o “coelhinho da Páscoa” concorde comigo.
>>>
>>> Abraço.
>>>
>>> Cláudio.
>>>
>>>
>>>
>>> *De:* owner-ob...@mat.puc-rio.br [mailto:owner-ob...@mat.puc-rio.br] *Em
>>> nome de *Douglas Oliveira de Lima
>>> *Enviada em:* domingo, 1 de abril de 2018 17:25
>>> *Para:* obm-l@mat.puc-rio.br
>>> *Assunto:* [obm-l] Geometria plana
>>>
>>>
>>>
>>> Olá amigos, pra quem gosta de geometria plana, compartilhando aqui uma
>>> questão do coelhinho da páscoa que achei legal.
>>>
>>>
>>>
>>> 1) Em um quadrado ABCD de lado unitário tomam-se os pontos P e Q sobre
>>> os lados AB e AD respectivamente, de modo que o perímetro do triângulo APQ
>>> seja igual a 2. Calcule a medida do ângulo  PCQ.
>>>
>>>
>>>
>>> Um abraço
>>>
>>>
>>>
>>> Douglas Oliveira.
>>>
>>>
>>> --
>>> Esta mensagem foi verificada pelo sistema de antiv�rus e
>>> acredita-se estar livre de perigo.
>>>
>>>
>>> 
>>>  Livre
>>> de vírus. www.avast.com
>>> .
>>>
>>> <#m_-5665104085043217724_m_2647765965440199561_m_3844324294932745576_DAB4FAD8-2DD7-40BB-A1B8-4E2AA1F9FDF2>
>>>
>>> --
>>> Esta mensagem foi verificada pelo sistema de antivírus e
>>> acredita-se estar livre de perigo.
>>>
>>
>> --
>> Esta mensagem foi verificada pelo sistema de antivírus e
>> acredita-se estar livre de perigo.
>>
>
>
> --
> Esta mensagem foi verificada pelo sistema de antivírus e
> acredita-se estar livre de perigo.
>

-- 
Esta mensagem foi verificada pelo 

[obm-l] Re: [obm-l] Ajuda em geometria e álgebra.

2018-03-13 Por tôpico Julio César Saldaña Pumarica
Seja P o ponto de DC tal que AP=AC (portanto igual ao BD). Calculando
alguns ângulos: APc=48 e PAD=18.

Seja O o circuncentro do triângulo APD, então OD=OP=OA, e como ADB=30 então
POA=2x30=60. Concluimos que o triângulo POA é equilátero. Calculando alguns
ângulos: ODA=42

Notando que OD=OB podemos concluir que OBD=DBO=36. Estique BO e desenh o
segmento AT perpendicular a BO (T está na prolongação de BO). Observe que
os triângulos ATO e APM (onde M é o meio de PC) são iguais e portanto AM=AT.

Finalmente os triângulo BAT e BAM são iguais e daí ABT=ABD=36/2=18

2018-03-13 20:14 GMT-03:00 Douglas Oliveira de Lima <
profdouglaso.del...@gmail.com>:

> Olá amigos, não consigo fazer esse problema por construção, já fiz por lei
> dos senos e
> pelo geogebra e deu 18 graus.
>
> Eis o problema:
>
> 6 Seja D um ponto sobre o lado BC de um triângulo ABC. Supondo que, AC=BD
> e o ângulo  ADC=30 graus e ACB= 48 graus , determine  a medida do ângulo
> ABC.
>
>
>
> Qualquer ajuda será bem vinda.
>
> Abraço do
>
> Douglas Oliveira.
>
> --
> Esta mensagem foi verificada pelo sistema de antivírus e
> acredita-se estar livre de perigo.

-- 
Esta mensagem foi verificada pelo sistema de antiv�rus e
 acredita-se estar livre de perigo.



[obm-l] Re: [obm-l] Geometria plana

2017-07-08 Por tôpico Julio César Saldaña



Note que os triângulos ABD e BCE são equivalentes (mesma área).
Baseado nisso podemos concluir que BE=AD; pois areas iguais e alturas iguais
implica bases iguais.

Então os triângulos ABD e BCE além de equivalente são congruentes (L.A.L.).
Portanto 

[obm-l] Re: [obm-l] Geometria plana (Ajuda)

2017-06-29 Por tôpico Julio César Saldaña



Aproveitando que APC é isósceles (pois CA=CP), eu desenhei a altura CH, então 
AH=HP e anguloACH=anguloHCP=20; mas como também anguloPCB=20, decidi desenhar a 
perpendicular PN sobre BC, así temos PN=PH=HA. Aí não resisti e estiquei PN até 
K, onde NK=PN. Desenhei a linha BK também.


Nesse ponto me encontrei com um problema que já tinha resolvido faz algum tempo 
mas não lembro. Então ensaiei outra solução. O problema é provar que P é o 
circuncentro do triângulo ABK. Desta vez argumento assim: como anguloBAK=30 
então BK é igual ao circunrádio do triângulo ABK. Mas note que BK=BP (pois BC é 
mediatriz de PK). Então pronto, P encontrase na mediatriz de AK e também se 
encontra a uma distância de B igual ao circunradio, logo P é o circuncentro do 
triângulo ABK.


Com isso, o triângulo PBK é equilátero e portanto anguloPBN=30. Portanto 
anguloBEC=90 => EM é mediana relativa à hipotenusa do triângulo retângulo BEC. 
Resposta: anguloMEC=60


Espero não ter me engando, mas vou fazer um double check e também vou tentar 
lembrar a outra forma de provar que P é circuncentro de ABK


Julio Saldaña


-- Mensaje original ---
De : obm-l@mat.puc-rio.br
Para : obm-l@mat.puc-rio.br
Fecha : Wed, 28 Jun 2017 14:43:07 -0300
Asunto : Re: [obm-l] Geometria plana (Ajuda)

Opa desculpe, CF é ceviana que passa por P.

Em 28 de jun de 2017 11:05 AM, "Pedro José"  escreveu:


Bom dia!

O ponto F não foi definido, mas foram definidas duas medidas de ângulos
aos quais o ponto F pertence: BCF=20 graus e FCA=40 graus.
Não faltou definir o ponto F?

Sds,
PJMS

Em 28 de junho de 2017 09:15, Douglas Oliveira de Lima <
profdouglaso.del...@gmail.com> escreveu:


Olá meus amigos preciso de uma ajuda pra resolver a seguinte questão:

Num triângulo ABC , tracam-se as cevianas AD e BE, que se encontram no
ponto P, tal que BAD= 10 graus, DAC=70 graus, BCF=20 graus e FCA=40 graus,
traçando a ceviana BE que passa por P e o segmento de reta que une os
pontos E e M, sendo M ponto médio de BC, determinar o ângulo CME.

GRATO!!
Douglas Oliveira.

--
Esta mensagem foi verificada pelo sistema de antivírus e
acredita-se estar livre de perigo.




--
Esta mensagem foi verificada pelo sistema de antivírus e
acredita-se estar livre de perigo.


--
Esta mensagem foi verificada pelo sistema de antivírus e
acredita-se estar livre de perigo.




__
Si desea recibir, semanalmente, el Boletín Electrónico de la PUCP, ingrese a:
http://www.pucp.edu.pe/puntoedu/suscribete/


--
Esta mensagem foi verificada pelo sistema de antivírus e
acredita-se estar livre de perigo.

=
Instruções para entrar na lista, sair da lista e usar a lista em
http://www.mat.puc-rio.br/~obmlistas/obm-l.html
=


[obm-l] Re: [obm-l] Contra-positiva

2017-06-21 Por tôpico Julio César Saldaña



Eu acho que pode ter varias equivalencias, não apenas a que está colocando.

Para analissar isso eu definiria R= "x é diferente de 0" (acho que era isso, não 
sairam alguns símbolos no seu email).


Nesse caso a primera afirmação sería:

R -> (P->Q)

E algumas equivalentes seriam:

i) R -> (~Q->~P) ; Isto responderia sua pergunta: o "diferente de 0" não deve 
mudar


ii) ~(P->Q) -> ~R ; Neste caso o "diferente de 0" sim deveria mudar par "igual a 
0"


O que acha dessa forma de ver?

Obrigado

Julio
Julio Saldaña


-- Mensaje original ---
De : obm-l@mat.puc-rio.br
Para : obm-l@mat.puc-rio.br
Fecha : Wed, 21 Jun 2017 17:11:00 -0300
Asunto : [obm-l] Contra-positiva

Olá amigos, boa tarde a todos.

Eu consigo provar que P(x)->Q(x) para todo x≠0, esta proposição é
equivalente a provar que
~Q(x)->~P(x) para todo x≠0?A minha dúvida é se esse x diferente de zero
passa a ser x=0 ou continua sendo x diferente de zero na contra-positiva.O
que eu penso que é o certo é que se P(x) implica Q(x) para todo x diferente
de zero, então, isto é equivalente a dizer que a negação de Q(x) implica a
negação de P(x) para todo x≠0, qual é a forma correta?


Desde já agradeço o auxílio amigos,
Israel Meireles Chrisostomo.

--
Esta mensagem foi verificada pelo sistema de antivírus e
acredita-se estar livre de perigo.




__
Si desea recibir, semanalmente, el Boletín Electrónico de la PUCP, ingrese a:
http://www.pucp.edu.pe/puntoedu/suscribete/


--
Esta mensagem foi verificada pelo sistema de antivírus e
acredita-se estar livre de perigo.

=
Instruções para entrar na lista, sair da lista e usar a lista em
http://www.mat.puc-rio.br/~obmlistas/obm-l.html
=


[obm-l] Re: [obm-l] Problema Geometria

2017-05-09 Por tôpico Julio César Saldaña



Solução um pouco longa:

-  PB=PE

- ABEC é inscritível => 
triângulo MEP = triângulo PEC (LAL). Por tanto 

[obm-l] Re: [obm-l] Geometria

2017-04-17 Por tôpico Julio César Saldaña



Imagino que D esteja sobre BC. Se for esse o caso:

ABD e AEC são congruentes.

Ángulo BAD = ángulo ECA e por isso ângulo DFC = 60, logo BEFD é inscritível.

EB = 2. BD e como ângulo B = 60 então ângulo EDB=90.

Como BEFD é inscritível então ângulo BFE=90 e finalmente ângulo BFC=90

Julio Saldaña


-- Mensaje original ---
De : obm-l@mat.puc-rio.br
Para : obm-l@mat.puc-rio.br
Fecha : Mon, 17 Apr 2017 11:55:34 -0300
Asunto : [obm-l] Geometria

Bom dia a todos,

Gostaria de uma ajuda com o seguinte problema:

Dado um triângulo equilátero ABC, tal que sobre o lado AB tenhamos um ponto
E e sobre o AC tenhamos um ponto D, com AE=BD=AB/3. Se as cevianas AD e CE
intersectam  no ponto F, qual a medida do ângulo BFC?

Grato pela atenção.

Abraços,

Marcelo




"Matemática é o alfabeto com o qual Deus escreveu o Universo"
Galileu Galilei

--
Esta mensagem foi verificada pelo sistema de antivírus e
acredita-se estar livre de perigo.




__
Si desea recibir, semanalmente, el Boletín Electrónico de la PUCP, ingrese a:
http://www.pucp.edu.pe/puntoedu/suscribete/


--
Esta mensagem foi verificada pelo sistema de antivírus e
acredita-se estar livre de perigo.

=
Instruções para entrar na lista, sair da lista e usar a lista em
http://www.mat.puc-rio.br/~obmlistas/obm-l.html
=


[obm-l] Re: {Disarmed} Re: [obm-l] Re: {Disarmed} Re: [obm-l] Re: [obm-l] Geometria plana

2015-03-04 Por tôpico Julio César Saldaña



Muito boa, vou guardar.

Obrigado

Julio Saldaña


-- Mensaje original ---
De : obm-l@mat.puc-rio.br
Para : obm-l@mat.puc-rio.br
Fecha : Tue, 3 Mar 2015 22:13:54 -0300
Asunto : {Disarmed} Re: [obm-l] Re: {Disarmed} Re: [obm-l] Re: [obm-l] Geometria
plana

Vou compartilhar uma para termos soluções alternativas:
1)Circunscreva um círculo ao triângulo ABC.
2) Prolongue AD até tocar o círculo em F.
3)Trace de B para F e de C para F.
4)Encontre BFA=AFC=90-(BAC)/2
5)Como FA é uma bissetriz teremos BF=2FC.
6)Como BEF é isósceles,  tome um ponto M médio  de BF e Trace EM.
7)Os triângulos EMF e EFC são congruentes,  assim FEC=(BAC) /2

Douglas Oliveira.
Em 03/03/2015 16:04, Julio César Saldaña saldana...@pucp.edu.pe
escreveu:




Isso mesmo, M �ponto medio de BE,

obrigado


Julio Salda馻


-- Mensaje original ---
De : obm-l@mat.puc-rio.br
Para : obm-l@mat.puc-rio.br
Fecha : Tue, 3 Mar 2015 15:33:26 -0300
Asunto : {Disarmed} Re: [obm-l] Re: [obm-l] Geometria plana
Bela solu莽茫o.

houve s贸 um pequeno erro de digita莽茫o : M 茅 ponto m茅dio de BE, ok ?

Pacini

Em 3 de mar莽o de 2015 11:53, Julio C茅sar Salda帽a saldana...@pucp.edu.pe
escreveu:



 Fiz assim, mas cuidado, costumo me equivocar muito. Podem verificar?

 Notar que ABE=EAC.

 Seja N de AC tal que DN 茅 paralelo 脿 AB, ent茫o DN=NC e AN=2.DN

 Como os tri芒ngulos ABE e ADN s茫o semelhantes ent茫o BE=2.AE

 Seja M o ponto medio de AE, ent茫o BM=ME=AE, e AME=MAE=40.

 Os tri芒ngulos BAM e EAC s茫o congruentes, por tanto igualamos 芒ngulos
 externos
 respectivos: DEC=40.


 Julio Salda帽a


 -- Mensaje original ---
 De : obm-l@mat.puc-rio.br
 Para : obm-l@mat.puc-rio.br
 Fecha : Mon, 2 Mar 2015 09:23:52 -0300
 Asunto : [obm-l] Geometria plana
 Ol脙隆,  bom dia quero compartilhar uma boa quest脙拢o de geometria

com os

 senhores,
 Q1) Num tri脙垄ngulo is脙鲁sceles ABC com AB=AC,  toma-se um ponto D no
lado
 BC
 de forma que BD=2CD e um ponto E em AD tal que os 脙垄ngulos BAC e BED
sejam
 iguais a 80 graus,   encontrar o valor do 脙垄ngulo DEC.
 
 Douglas Oliveira.
 
 --
 Esta mensagem foi verificada pelo sistema de antiv铆rus e
  acredita-se estar livre de perigo.
 


 __
 Si desea recibir, semanalmente, el Bolet铆n Electr贸nico de la PUCP,
ingrese
 a:
 http://www.pucp.edu.pe/puntoedu/suscribete/


 --
 Esta mensagem foi verificada pelo sistema de antiv铆rus e
  acredita-se estar livre de perigo.


=
 Instru莽玫es para entrar na lista, sair da lista e usar a lista em
 http://www.mat.puc-rio.br/~obmlistas/obm-l.html

=


--
Esta mensagem foi verificada pelo sistema de antiv韗us e
 acredita-se estar livre de perigo.



__
Si desea recibir, semanalmente, el Bolet韓 Electr髇ico de la PUCP, ingrese a:
http://www.pucp.edu.pe/puntoedu/suscribete/


--
Esta mensagem foi verificada pelo sistema de antiv韗us e
 acredita-se estar livre de perigo.

=
Instru珲es para entrar na lista, sair da lista e usar a lista em
http://www.mat.puc-rio.br/~obmlistas/obm-l.html
=



--
Esta mensagem foi verificada pelo sistema de antivírus e
acredita-se estar livre de perigo.




__
Si desea recibir, semanalmente, el Boletín Electrónico de la PUCP, ingrese a:
http://www.pucp.edu.pe/puntoedu/suscribete/


--
Esta mensagem foi verificada pelo sistema de antivírus e
acredita-se estar livre de perigo.

=
Instruções para entrar na lista, sair da lista e usar a lista em
http://www.mat.puc-rio.br/~obmlistas/obm-l.html
=


[obm-l] Re: [obm-l] Geometria plana

2015-03-03 Por tôpico Julio César Saldaña



Fiz assim, mas cuidado, costumo me equivocar muito. Podem verificar?

Notar que ABE=EAC.

Seja N de AC tal que DN é paralelo à AB, então DN=NC e AN=2.DN

Como os triângulos ABE e ADN são semelhantes então BE=2.AE

Seja M o ponto medio de AE, então BM=ME=AE, e AME=MAE=40. 


Os triângulos BAM e EAC são congruentes, por tanto igualamos ângulos externos
respectivos: DEC=40.


Julio Saldaña


-- Mensaje original ---
De : obm-l@mat.puc-rio.br
Para : obm-l@mat.puc-rio.br
Fecha : Mon, 2 Mar 2015 09:23:52 -0300
Asunto : [obm-l] Geometria plana

Olá,  bom dia quero compartilhar uma boa questão de geometria com os
senhores,
Q1) Num triângulo isósceles ABC com AB=AC,  toma-se um ponto D no lado BC
de forma que BD=2CD e um ponto E em AD tal que os ângulos BAC e BED sejam
iguais a 80 graus,   encontrar o valor do ângulo DEC.

Douglas Oliveira.

--
Esta mensagem foi verificada pelo sistema de antivírus e
acredita-se estar livre de perigo.




__
Si desea recibir, semanalmente, el Boletín Electrónico de la PUCP, ingrese a:
http://www.pucp.edu.pe/puntoedu/suscribete/


--
Esta mensagem foi verificada pelo sistema de antivírus e
acredita-se estar livre de perigo.

=
Instruções para entrar na lista, sair da lista e usar a lista em
http://www.mat.puc-rio.br/~obmlistas/obm-l.html
=


[obm-l] Re: {Disarmed} Re: [obm-l] Re: [obm-l] Geometria plana

2015-03-03 Por tôpico Julio César Saldaña



Isso mesmo, M é ponto medio de BE,

obrigado


Julio Saldaña


-- Mensaje original ---
De : obm-l@mat.puc-rio.br
Para : obm-l@mat.puc-rio.br
Fecha : Tue, 3 Mar 2015 15:33:26 -0300
Asunto : {Disarmed} Re: [obm-l] Re: [obm-l] Geometria plana

Bela solução.

houve só um pequeno erro de digitação : M é ponto médio de BE, ok ?

Pacini

Em 3 de março de 2015 11:53, Julio César Saldaña saldana...@pucp.edu.pe
escreveu:




Fiz assim, mas cuidado, costumo me equivocar muito. Podem verificar?

Notar que ABE=EAC.

Seja N de AC tal que DN é paralelo à AB, então DN=NC e AN=2.DN

Como os triângulos ABE e ADN são semelhantes então BE=2.AE

Seja M o ponto medio de AE, então BM=ME=AE, e AME=MAE=40.

Os triângulos BAM e EAC são congruentes, por tanto igualamos ângulos
externos
respectivos: DEC=40.


Julio Saldaña


-- Mensaje original ---
De : obm-l@mat.puc-rio.br
Para : obm-l@mat.puc-rio.br
Fecha : Mon, 2 Mar 2015 09:23:52 -0300
Asunto : [obm-l] Geometria plana
Olá,  bom dia quero compartilhar uma boa questão de geometria com os
senhores,
Q1) Num triângulo isósceles ABC com AB=AC,  toma-se um ponto D no lado
BC
de forma que BD=2CD e um ponto E em AD tal que os ângulos BAC e BED sejam
iguais a 80 graus,   encontrar o valor do ângulo DEC.

Douglas Oliveira.

--
Esta mensagem foi verificada pelo sistema de antivírus e
 acredita-se estar livre de perigo.



__
Si desea recibir, semanalmente, el Boletín Electrónico de la PUCP, ingrese
a:
http://www.pucp.edu.pe/puntoedu/suscribete/


--
Esta mensagem foi verificada pelo sistema de antivírus e
 acredita-se estar livre de perigo.

=
Instruções para entrar na lista, sair da lista e usar a lista em
http://www.mat.puc-rio.br/~obmlistas/obm-l.html
=



--
Esta mensagem foi verificada pelo sistema de antivírus e
acredita-se estar livre de perigo.




__
Si desea recibir, semanalmente, el Boletín Electrónico de la PUCP, ingrese a:
http://www.pucp.edu.pe/puntoedu/suscribete/


--
Esta mensagem foi verificada pelo sistema de antivírus e
acredita-se estar livre de perigo.

=
Instruções para entrar na lista, sair da lista e usar a lista em
http://www.mat.puc-rio.br/~obmlistas/obm-l.html
=


[obm-l] Re: [obm-l] Triângulo e circunferências

2014-10-10 Por tôpico Julio César Saldaña



Bom, boa solução, não garanto. Ao menos da para encontrar o raio:

Que tal um teorema da bisectriz:

3 / 5 = R /(4-R)





Julio Saldaña


-- Mensaje original ---
De : obm-l@mat.puc-rio.br
Para : obm-l@mat.puc-rio.br
Fecha : Thu, 9 Oct 2014 21:51:28 -0300
Asunto : [obm-l] Triângulo e circunferências

Olá meus caros...depois de uma longa temporada em off na lista vou postar
uma perguntinha...

Traçamos o círculo T de centro O circunscrito a um triângulo ABC, retângulo
em A de catetos 3 cm e 4 cm. Encontre o raio do círculo W de centro O´,
tangente aos catetos de ABC e interiormente a T

Alguém tem um boa solução?

Abraços, Carlos Gomes.

--
Esta mensagem foi verificada pelo sistema de antivírus e
acredita-se estar livre de perigo.




__
Si desea recibir, semanalmente, el Boletín Electrónico de la PUCP, ingrese a:
http://www.pucp.edu.pe/puntoedu/suscribete/


--
Esta mensagem foi verificada pelo sistema de antivírus e
acredita-se estar livre de perigo.

=
Instruções para entrar na lista, sair da lista e usar a lista em
http://www.mat.puc-rio.br/~obmlistas/obm-l.html
=


[obm-l] Re: [obm-l] Triângulo e circunferências

2014-10-10 Por tôpico Julio César Saldaña



Bom, agora vou tentar uma solução que funcione, a anterior está errada.

Se P é o ponto de tangencia, Teorema da bisectriz sería:

PB/PA = (3-R)/R   (Supondo BA=3)

PC/PA = (4-R)/R

4 vezes a primeira mais 3 a segunda (para aproveitar Ptolomeo):

5. PA / PA = 4.(3-R)/R + 3.(4-R) / R

então R=2. Mas está muito enrolada essa solução, deve ter outra.


Julio Saldaña


-- Mensaje original ---
De : obm-l@mat.puc-rio.br
Para : obm-l@mat.puc-rio.br
Fecha : Thu, 9 Oct 2014 21:51:28 -0300
Asunto : [obm-l] Triângulo e circunferências

Olá meus caros...depois de uma longa temporada em off na lista vou postar
uma perguntinha...

Traçamos o círculo T de centro O circunscrito a um triângulo ABC, retângulo
em A de catetos 3 cm e 4 cm. Encontre o raio do círculo W de centro O´,
tangente aos catetos de ABC e interiormente a T

Alguém tem um boa solução?

Abraços, Carlos Gomes.

--
Esta mensagem foi verificada pelo sistema de antivírus e
acredita-se estar livre de perigo.




__
Si desea recibir, semanalmente, el Boletín Electrónico de la PUCP, ingrese a:
http://www.pucp.edu.pe/puntoedu/suscribete/


--
Esta mensagem foi verificada pelo sistema de antivírus e
acredita-se estar livre de perigo.

=
Instruções para entrar na lista, sair da lista e usar a lista em
http://www.mat.puc-rio.br/~obmlistas/obm-l.html
=


[obm-l] Re: [obm-l] Trigonometria.

2014-08-30 Por tôpico Julio César Saldaña



Olá, eu lembro ter rido uma aula de ângulos aproximados no cursinho de 
vestibular (no Peru). Para o triângulo pitagórico 20,21, e 29 os ângulos agudos 
mediam aproximadamente 41 e 49. Para o triângulo (não pitagórico) de catetos 1 e 
4 os ângulos agudos mediam 14 e 76.


Segundo isso o valor aproximado do ângulo fi seria 49+14=63, mas não está em 
nenhuma dal alternativas que você citou.


Segundo a minha calculadora um valor mais próximo sería 61.2

Julio Saldaña


-- Mensaje original ---
De : obm-l@mat.puc-rio.br
Para : obm-l@mat.puc-rio.br
Fecha : Sat, 30 Aug 2014 15:23:52 -0300
Asunto : [obm-l] Trigonometria.



Esse exercício caiu na primeira fase de uma Olimpíada. Três engrenagens
A, B e C estão assim dispostas. A é tangente à B e à C , mas B não é
tangente à C. Os raios das engrenagens são: A 28 cm , B 30 cm e C 22 cm.
Os centros das engrenagens são ligados por segmentos formando um
triângulo. A medida do ângulo do vértice que está em A mede fi ( letra
grega ). O ângulo cujo vértice está em B mede 41 graus. Quanto mede (
presumo aproximadamente) o ângulo fi? Estranho , pois tinha como
respostas os seguintes testes. : a)30 graus b) 40 graus c) 69,55 graus
d) 79, 55 graus e) 89,55 graus. Se alguém puder dar uma ajuda, agradeço
antecipadamente. Abraço. 


--
Esta mensagem foi verificada pelo sistema de antivírus e
acredita-se estar livre de perigo.




__
Si desea recibir, semanalmente, el Boletín Electrónico de la PUCP, ingrese a:
http://www.pucp.edu.pe/puntoedu/suscribete/


--
Esta mensagem foi verificada pelo sistema de antivírus e
acredita-se estar livre de perigo.

=
Instruções para entrar na lista, sair da lista e usar a lista em
http://www.mat.puc-rio.br/~obmlistas/obm-l.html
=


[obm-l] Re: [obm-l] Proposta de Metodo para Resolver Certos Problemas de Geometria

2014-07-08 Por tôpico Julio César Saldaña



Também existe o problema genérico do hexágono com A+C+E=360 em lugar de
A=C=E=120. Acho que até nome tem esse problema.

Julio Saldaña


-- Mensaje original ---
De : obm-l@mat.puc-rio.br
Para : obm-l@mat.puc-rio.br
Fecha : Tue, 8 Jul 2014 06:34:20 -0700
Asunto : Re: [obm-l] Proposta de Metodo para Resolver Certos Problemas de 
Geometria

Ola Rogério,

Eu conheço 2 soluções para este problema do pentágono :

1) trace as perpendiculares à diagonal EC, de A, ,B D e M (médio).  Brinque

com os triangulos que surgem.


2) trace AD e BD, considere P medio de AD e Q medio de BD; agora trabalhe com

os triangulos EPM e CQM.



Agora, vai um desafio que eu criei (adaptei):

Considere o hexágono convexo ABCDEF, onde A=C=E= 120 e AB=AF, BC=CD e ED=EF.
Determine o ângulo AEC. 



Se quisermos piorar um pouco as coisas, podemos colocar esse problema do

pentágono dentro desse desafio, e pedir para calcular o angulo que é formado
pela diferença dos ângulos de 45o, do caso do pentagono, para o ângulo desse
problema. Ou seja, para achar o resultado o caboclo teria que resolver 2
problemas muito bonitinhos envolvendo ângulo.


Abs
Felipe



Em Quarta-feira, 2 de Julho de 2014 15:33, Rogerio Ponce abrlw...@gmail.com

escreveu:




Ola' Felipe,
em relacao ao problema do pentagono que voce descreveu, talvez o enunciado do

problema estivesse incompleto, e o artificio de se levar a construcao a uma
situacao limite nao pudesse ser usado.


Ou seja, teriamos que, primeiramente, provar que o angulo CEM nao depende do

comprimento de AB.



Claro que para responder a uma questao de multipla escolha, vale o metodo -

tambem gosto dessas trapacas! -, mas se fosse uma questao discursiva bem
elaborada, certamente o  enunciado pediria para provar que o angulo seria
constante, independentemente das outras medidas do pentagono.



Entao, maos 'a obra!

Tente provar que o angulo CEM e' constante (e faca o favor de postar a solucao!)


Grande abraco,

Rogerio Ponce





2014-06-26 11:30 GMT-03:00 luiz silva luizfelipec...@yahoo.com.br:

Pessoal,

 
Descobri o seguinte teorema em um EXCELENTE livro de geometria peruano,

que um amigo comprou : dado um quadrilátero convexo qqer, construa 4 quadrados
externos ao mesmo, onde cada lado do quadrilatero seja um dos lados
de um dos quadrados. Una os centros dos quadrados opostos. O teorema diz que os
segmentos que unem os centros dos quadrados opostos tem a mesma medida e que o
angulo entre eles é de 90o.. Ainda não consegui demonstra-lo, porem acho que
fiz algo interessante :

- O teorema é válido para qqer media dos lados do quadrilátero. Então,

pequemos um dos lados do quadrilátero e dividamos por 2, assim, teremos um novo
quadrilátero, mas o teorema ainda é válido, faça o mesmo processo neste 
mesmo
lado, indefinidamente. Quando o número de iterações tender a infinito, a 
medida
de um dos lados do quadrilátero irá tender a zero (um ponto). 

 
Ou seja, o quadrilátero

tenderá a se tornar um triângulo, o quadrado referente a esse lado que foi
sendo dividido por dois, torna-se um ponto (o vértice desse triangulo),

porém, ainda

assim, o teorema ainda será válido; só que agora, ao invés de termos os 
centros
de 4 quadrados, teremos o centro de 3 quadrados e o vértice do traingulo.Eu

usei esse mesmo raciocínio para resolver um problema clássico de

ângulos : Dado um pentágono ABCDEF, onde EA=ED e E=90; CB=CD e C=90. Calcular 
o
ângulo CEM, onde M é medio de AB.Da mesma forma que acima, fui reduzindo a

medida do segmento AB, até o

mesmo se tornar um ponto. Quando isso ocorre, os lados se tornam iguais
(quadrado), o segmento EM tende a ser congruente a EA e EC tende a ser a 
diagonal
desse quadrado. Ou seja, o ângulo CEM = 45o.

Creio que possamos validar esse método através da geometria analítica: essas

“propriedades regulares” (cumprimento, ângulos entre retas) são função 
das
coordenadas dos pontos envolvidos no problema. E essas propriedades regulares

são descritas por funções contínuas em R.

Ainda não fiz, mas acho que não deve ser difícil demonstrar que esses 
resultados
são válidos mesmo quando um dos cumprimentos envolvidos no problema se reduz a
zero (a um ponto).

Creio que podemos aplicar esse mesmo método para um problema recente proposto

por um colega, problema este que é uma variação desse problema do pentágono.

 
Abs
Felipe
--
Esta mensagem foi verificada pelo sistema de antivírus e 
acredita-se estar livre de perigo. 


--
Esta mensagem foi verificada pelo sistema de antiv�us e 
acredita-se estar livre de perigo. 
--

Esta mensagem foi verificada pelo sistema de antivírus e
acredita-se estar livre de perigo.




__
Si desea recibir, semanalmente, el Boletín Electrónico de la PUCP, ingrese a:
http://www.pucp.edu.pe/puntoedu/suscribete/


--
Esta mensagem foi verificada pelo sistema de antivírus e

[obm-l] Re: [obm-l] Problema de geometria!

2014-07-07 Por tôpico Julio César Saldaña



Seja P o ponto da prolongação de CE tal que PBC=90 = PE/EC=3 = AP//FE

Seja Q o ponto da prolongação de AD tal que QBA=90 = AD/DQ=3 = QC//DF

Seja BQ=x = AB=x.sqrt(3)

Seja BC=y = BP=y.sqrt(3)

Então BQ/BC=x/y=AB/AP. E como QBC=ABP=90+B, então os triângulos QBP e ABP são
semelhantes. Com isso é facil mostrar que o ángulo entre QC e AP é 90, e como
esas linhas eram paralelas a DF e FE, então estas últimas sçao perpendiculares.

Também pela semelhança se prova: AP/QC=sqrt(3), e como DF e FE são terceiras
partes de QC e FE respetivamente = FE/DF=sqrt(3) então FDE=60.


Julio Saldaña


-- Mensaje original ---
De : obm-l@mat.puc-rio.br
Para : obm-l@mat.puc-rio.br
Fecha : Sun, 6 Jul 2014 18:02:13 -0300
Asunto : Re: [obm-l] Problema de geometria!

OI Douglas ,

Pensando neste problema, se usar a lei dos cossenos nos triângulos FCE, AFD
e DBE e usando o fato de que cos(90+B)= -senB ( não é muito trabalhoso);
deixando na forma de quadrados não é difícil de concluir que 4 FE^2= ED^2
e que 4DF^2 = 3ED^2 ; ou seja o triângulo EFD é retângulo e que que os
ângulos pedidos são 90º e 30º , ok ?

Pelo visto vc está querendo uma solução com algum traçado mágico, não é
verdade?
Estarei pensando, ok  Douglas! Estou desconfiado que deve ter alguma coisa
com o Teorema de Napoleão ...

Vamos tentar, pois deve ser assaz interessante tal traçado.

Abraços

Carlos Victor


Em 13 de junho de 2014 16:09, Douglas Oliveira de Lima 
profdouglaso.del...@gmail.com escreveu:


Olá caros amigos, me encontro mais uma vez com um pequeno problema de
geometria no qual estou com uma solução muito absurda(muito trabalho
braçal), gostaria de uma ajuda com outras soluções, desde já agradeço a
colaboração dos senhores.

PROBLEMA:
Considere um triângulo ABC, são construídos externamente os triângulos ADB
e BCE de forma que ADB=BEC=90 GRAUS E DAB=EBC=30 graus. No segmento AC
marca-se o ponto F tal que AF=3FC. Calcular os ângulos DFE e FDE.

Douglas Oliveira de Lima

--
Esta mensagem foi verificada pelo sistema de antivírus e
acredita-se estar livre de perigo.


--
Esta mensagem foi verificada pelo sistema de antivírus e
acredita-se estar livre de perigo.




__
Si desea recibir, semanalmente, el Boletín Electrónico de la PUCP, ingrese a:
http://www.pucp.edu.pe/puntoedu/suscribete/


--
Esta mensagem foi verificada pelo sistema de antivírus e
acredita-se estar livre de perigo.

=
Instruções para entrar na lista, sair da lista e usar a lista em
http://www.mat.puc-rio.br/~obmlistas/obm-l.html
=


[obm-l] Re: [obm-l] Re: [obm-l] Re: [obm-l] Mais duas questões excelentes de geometria!!!

2014-05-24 Por tôpico Julio César Saldaña



Desculpe prof. Renato, numa primeira busca não consegui achar o pdf do Shariguin 
em internet. Poderia sugerir algum link?


Obrigado

Julio Saldaña


-- Mensaje original ---
De : obm-l@mat.puc-rio.br
Para : obm-l@mat.puc-rio.br
Fecha : Sat, 24 May 2014 09:39:50 -0300
Asunto : Re: [obm-l] Re: [obm-l] Re: [obm-l] Mais duas questões excelentes de 
geometria!!!

Douglas,

O problema 2 aparece resolvido no livro de Geometria Espacial do Shariguin 

(Mir). É fácil achar o pdf dele na internet.


Att, Renato Madeira.

Em 23/05/2014, às 17:37, Douglas Oliveira de Lima 

profdouglaso.del...@gmail.com escreveu:


Tenho sim , assim que tiver um tempinho pra tirar xerox eu te comunico.


Em 23 de maio de 2014 16:42, Mauricio de Araujo 

mauricio.de.ara...@gmail.com escreveu:

​boa tarde!!

você ainda tem essa apostila? Se sim, poderia enviar uma cópia para 

mim? levante os custos por favor.


M.​


2014-05-15 17:24 GMT-03:00 Douglas Oliveira de Lima 

profdouglaso.del...@gmail.com:
Quando estava estudando para a prova do colégio naval em 1997, no 
colégio e curso tamandare da ilha do governador(Diretores Orozimbo e Oswaldo) 
me deparei com uma apostila em xerox escrita pelos professores Carlos Victor e 
Eduardo Mauro, que tinham provas resolvidas do colégio naval e no final da 
apostila tinham questões propostas por eles, e uma delas que gostei muito e 
fiz na época(sem uso de calculo)
 quero compartilhar com os senhores, e também outro problema numero 2 
que ainda não consegui fazer mas estou tentando. A saber so fui conseguir a 
apostila no final do curso perto da prova assim continuei estudando ate que 
passei em primeiro lugar geral de matemática(nesta época um ano atras 
pensei que não passaria numa prova dessas nunca).


PROBLEMA 1)(Proposta por Carlos Victos e Eduardo Mauro ) Dado um quarto de 
circulo AOB, de raio OA, prolonga-se o raio OA, e pelo ponto B, traca-se uma 
perpendicular ao raio OB.


a) Tracar a este quarto de circulo uma tangente MN, tal que a área do 

trapézio BMNO seja igual a uma área dada m^2.

b) Ache o minimo da area do trapezio.

PROBLEMA 2)Seja um cubo de aresta a. Seja N um ponto na diagonal de uma 
face lateral, M um ponto no círculo que se encontra no plano da base com 
centro no centro da base e raio (5/12)a  Encontre o menor valor da medida do 
segmento MN.


--
Esta mensagem foi verificada pelo sistema de antivírus e 
acredita-se estar livre de perigo.




--
Abraços

oɾnɐɹɐ ǝp oıɔıɹnɐɯ


--
Esta mensagem foi verificada pelo sistema de antivírus e 
acredita-se estar livre de perigo.



--
Esta mensagem foi verificada pelo sistema de antivírus e 
acredita-se estar livre de perigo.


--
Esta mensagem foi verificada pelo sistema de antivírus e
acredita-se estar livre de perigo.




__
Si desea recibir, semanalmente, el Boletín Electrónico de la PUCP, ingrese a:
http://www.pucp.edu.pe/puntoedu/suscribete/


--
Esta mensagem foi verificada pelo sistema de antivírus e
acredita-se estar livre de perigo.

=
Instruções para entrar na lista, sair da lista e usar a lista em
http://www.mat.puc-rio.br/~obmlistas/obm-l.html
=


[obm-l] Re: [obm-l] Geometria Plana

2014-05-23 Por tôpico Julio César Saldaña



Seja M a interseção de BC com a circunferência, então AM é altura. Então MEC =
MAC = EBC.

Devido a ter os mesmos ângulos, os triângulos BEC e MEC são semeljantes, então
EC / 1 = 2/ EC, por tanto EC = sqrt(2).

Julio Saldaña


-- Mensaje original ---
De : obm-l@mat.puc-rio.br
Para : obm-l@mat.puc-rio.br
Fecha : Fri, 23 May 2014 00:46:24 -0300
Asunto : [obm-l] Geometria Plana

Olá,
Alguém pode me ajudar no exercício que segue

Seja ABC um triângulo isósceles, com AB=AC. Com centro no ponto médio de
AC, traça-se uma circunferência de diâmetro AB. Por B, traçamos uma altura
do triângulo, que intercepta a circunferência em E. Sabendo que BC=2,
determine o valor de CE.

Desde já, agradeço pela devida atenção

--
Esta mensagem foi verificada pelo sistema de antivírus e
acredita-se estar livre de perigo.




__
Si desea recibir, semanalmente, el Boletín Electrónico de la PUCP, ingrese a:
http://www.pucp.edu.pe/puntoedu/suscribete/


--
Esta mensagem foi verificada pelo sistema de antivírus e
acredita-se estar livre de perigo.

=
Instruções para entrar na lista, sair da lista e usar a lista em
http://www.mat.puc-rio.br/~obmlistas/obm-l.html
=


[obm-l] Re: [obm-l] Ajuda em geometria(Ja foram resolvidas por inspeção usando trigonometria)

2014-05-16 Por tôpico Julio César Saldaña



Seja Q o ponto de AC tal que PQ=QA.

Seja T o ponto de AB tal que APT=20.

Analizando o triângulo ATP e o ponto Q: ângulo externo em T = 50, angulo AQP =
100 (= 2 x 50), e QA=QP, conclusão Q é circuncentro de ATP. Então QT=QA=QP
(circunradio). Então Triângulo TQP é equilátero, então  TP=TQ.

COm isso tudo, Os triângulos TPC e TQC são idênticos, portanto PCT=10. E como o
ângulo PCT tambem é 10 e además TC=BC (pois BTC=80, esquecia isso). Ouseja os
triângulos BCP e TCP são idênticos, por tanto PB=PT e então x=50

Julio Saldaña


-- Mensaje original ---
De : obm-l@mat.puc-rio.br
Para : obm-l@mat.puc-rio.br
Fecha : Thu, 15 May 2014 16:58:44 -0300
Asunto : [obm-l] Ajuda em geometria(Ja foram resolvidas por inspeção usando
trigonometria)

Ola meus caros amigos, desenhando aqui pelo geogebra acabei criando uma
bela questão de geometria, do qual consegui por inspecao resolve-la através
de trigonometria pela lei dos senos, porem fiquei muito curioso para saber
se existe alguma solução por geometria euclidiana plana, estarei tentando,
mas vim aqui compartilhar com voces, se puderem agradeço desde ja.Um abraço
do Douglas Oliveira.

Problema: Seja um triângulo ABC com ângulos BAC=70 graus, ACB=30; dado um
ponto interno P such that BAP=30 e BCP=10, encontrar o angulo ABP.

--
Esta mensagem foi verificada pelo sistema de antivírus e
acredita-se estar livre de perigo.




__
Si desea recibir, semanalmente, el Boletín Electrónico de la PUCP, ingrese a:
http://www.pucp.edu.pe/puntoedu/suscribete/


--
Esta mensagem foi verificada pelo sistema de antivírus e
acredita-se estar livre de perigo.

=
Instruções para entrar na lista, sair da lista e usar a lista em
http://www.mat.puc-rio.br/~obmlistas/obm-l.html
=


[obm-l] Re: [obm-l] Problema de Geometria

2013-04-28 Por tôpico Julio César Saldaña



F é baricentro do triángulo ADB, logo FO=b/3, então FE=a/2-b/3

Julio Saldaña


-- Mensaje original ---
De : obm-l@mat.puc-rio.br
Para : obm-l@mat.puc-rio.br
Fecha : Sun, 28 Apr 2013 18:42:50 -0300
Asunto : Re: [obm-l] Problema de Geometria

Olá Raphael,
Pense no seguinte :

1) Trace OC
2) Trace BD
3) Conclua que BD é o dobro de OC.
4) Denomine EF = x
5) Faça  a semelhança de OCF com BFD e determine x , ok ?

Abraços

Carlos  Victor


Em 28 de abril de 2013 18:19, Raphael Feijao
raphaelfei...@hotmail.comescreveu:


O segmento AB é o diametro de uma circunferencia de centro O. Toma-se um
ponto C desse círculo e prolonga-se o segmento AC de um segmento CD igual a
AC. O segmento OD corta a circunferencia em E e corta o segmento BC em F.
Se AB=a e OD=b. Calcule EF.




__
Si desea recibir, semanalmente, el Boletín Electrónico de la PUCP, ingrese a:
http://www.pucp.edu.pe/puntoedu/suscribete/

=
Instruções para entrar na lista, sair da lista e usar a lista em
http://www.mat.puc-rio.br/~obmlistas/obm-l.html
=


[obm-l] Re: [obm-l] Geometria

2013-03-13 Por tôpico Julio César Saldaña



Vamos supor que AB=c é o maior dos lados.

Se desde o vértice C desenhamos um diámtro CP, teremos que PA**2 = 4R**2-b**2, e
também que PB**2 = 4R**2-a**2, logo, no triángulo APB temos que a soma dos
quadrados de dois lados é: PA**2+PB**2=8R**2-a**2-b**2, que segundo dado do
problema é igual a c**2, ou seja: PA**2+PB**2 = AB**2, logo o triángulo APB é
retângulo e portanto AB é diâmetro, ouseja o ABC também é retángulo.

cofesso que não estou convencido com a minha solução, acho que é incompleta.

Julio Saldaña


-- Mensaje original ---
De : obm-l@mat.puc-rio.br
Para : obm-l@mat.puc-rio.br
Fecha : Wed, 13 Mar 2013 18:25:40 +
Asunto : [obm-l] Geometria

Saja um triângulo cujos lados medem a,b e c e R o raio da circunferência

circunscrita.Mostre que a^2 + b^2 + c^2 = 8R^2 se,e somente se,o triângulo é
retângulo. Se o triangulo é retangulo,considerando a  = b  c,temos que a^2 +
b^2 = c^2a^2 + b^2 + c^2= 2c^2 e,como c = 2R,segue quea^2 + b^2 + c^2 = 2.(2R)^2
= 8R^2 Estou tentando a segunda parte da demonstração e não sai. :  		 	   		  

--
Esta mensagem foi verificada pelo sistema de antivírus e
acredita-se estar livre de perigo.




__
Si desea recibir, semanalmente, el Boletín Electrónico de la PUCP, ingrese a:
http://www.pucp.edu.pe/puntoedu/suscribete/

=
Instruções para entrar na lista, sair da lista e usar a lista em
http://www.mat.puc-rio.br/~obmlistas/obm-l.html
=


[obm-l] Re: [obm-l] problema de geometria difícil

2012-12-27 Por tôpico Julio César Saldaña



Bom, aqui tem uma solução para o problema 1 que emprega conceitos de
quadrilátero cíclico. Acho que já postei uma que só usa congruência de
triângulos, vou procurar.


Primeiro vamos provar que CE=AB. Seja M o ponto meio de AB, então ACM=MCB=10

Seja P o ponto de interseção de CM e BD. Então APB é um triângulo equilátero (se
desejam posso argumentar melhor isso em outro email). Finalmente o triângulo APC
é congruente ao triângulo AEC (caso ALA), em conclusão CE=AP=AB.


Bem, sabendo agora que CE=AB (apaguemos as linhas auxiliares trazadas para
provar aquilo), sabendo además que BD=DC, vamos procurar um triângulo congurente
ao triângulo CDE. Isso é provocativo pois temos dois pares de lados iguais CE=AB
e CD=DB. Seja Q o ponto de AE tal que  BQ=BA, então BAQ=AQB=70, então ABQ=40,
então QBD=20, então pronto!, temos o triângulo QBD congruente ao triângulo CDE
(caso LAL). Varias conclusões tiramos disso, por exemplo QD=DE, e mais
interessante ainda: DEC=DQB, ou seja o quadrilátero BQDE é cíclico, por tanto
DEQ=x=DBQ=20.

Julio

Julio Saldaña


-- Mensaje original ---
De : obm-l@mat.puc-rio.br
Para : obm-l@mat.puc-rio.br
Fecha : Fri, 21 Dec 2012 00:21:21 -0200
Asunto : [obm-l] problema de geometria difícil

Pessoal, alguém conhece uma solução para o primeiro problema da página a
seguir???

http://thinkzone.wlonk.com/MathFun/Triangle.htm

Ele é chamada de problema mais difícil com geometria fácil do mundo.

O segundo problema é famoso, mas o primeiro...



__
Si desea recibir, semanalmente, el Boletín Electrónico de la PUCP, ingrese a:
http://www.pucp.edu.pe/puntoedu/suscribete/

=
Instruções para entrar na lista, sair da lista e usar a lista em
http://www.mat.puc-rio.br/~obmlistas/obm-l.html
=


[obm-l] Re: [obm-l] problema de geometria difícil

2012-12-27 Por tôpico Julio César Saldaña



Bem agora envio uma outra solução que não precisa do quadrilátero cíclico.

Vou aproveitar o fato já provado que CE=AB. Seja T o ponto de AD tal que AB=BT,
então TAB=ATB=80, então TBD=40, então BT=TD (pois TDB=TBD). Notemos que
TBC=60, assim sendo sinto uma enorme força para localizar o ponto N de BE tal
que BN=BT, então o triângulo TNB é equilátero, então TN=TD, então TND=TDN=70
(pois DTN=100-60=40), então NDB=30. Finalmente notemos que o triângulo DBN é
congruente ao triângulo DCE (caso LAL, recordemos que já provamos que CE=AB, e
agoram sabemos que AB=DN, además já sabiamos também que BD=DC). Bem, como
conclusaõ da congruência, CDB=NDB=30, então DEA=x=30-10=20.


Julio Saldaña


-- Mensaje original ---
De : obm-l@mat.puc-rio.br
Para : obm-l@mat.puc-rio.br
Fecha : Fri, 21 Dec 2012 00:21:21 -0200
Asunto : [obm-l] problema de geometria difícil

Pessoal, alguém conhece uma solução para o primeiro problema da página a
seguir???

http://thinkzone.wlonk.com/MathFun/Triangle.htm

Ele é chamada de problema mais difícil com geometria fácil do mundo.

O segundo problema é famoso, mas o primeiro...



__
Si desea recibir, semanalmente, el Boletín Electrónico de la PUCP, ingrese a:
http://www.pucp.edu.pe/puntoedu/suscribete/

=
Instruções para entrar na lista, sair da lista e usar a lista em
http://www.mat.puc-rio.br/~obmlistas/obm-l.html
=


[obm-l] Re: [obm-l] Geometria( Questão di ficil(?))

2012-11-01 Por tôpico Julio César Saldaña



Esta vez va en español,

por el punto C trace una recta paralela a AD. Sea P el punto de intersección de
esa recta con la recta que contiene a los puntos A y B. Entonces por el teorema
de thales: AB/BP = BD/BC, es decir ABxBC = BDxBP, por lo tanto los triángulos
ABC y PBD son equivalentes. Si M es el punto medio de BP, el área del triángulo
BMD es la mitad del área del triángulo BPD, es decir, será igual a la mitad del
área del triángulo ABC. Por lo tanto la recta buscada es MD.

Si alguien pudiera traducir?


Julio Saldaña


-- Mensaje original ---
De : obm-l@mat.puc-rio.br
Para : obm-l@mat.puc-rio.br
Fecha : Thu, 1 Nov 2012 11:43:49 +
Asunto : [obm-l] Geometria( Questão di ficil(?))





Dado um triangulo ABC e D um ponto do lado BC,determine a reta passando por D

que separa op triangulo em duas regiões de igual área. Se D for ponto
médio,claro que a reta procurada contem a mediana de BC.No mais...é socorro
mesmo. 		 	   		   


__
Si desea recibir, semanalmente, el Boletín Electrónico de la PUCP, ingrese a:
http://www.pucp.edu.pe/puntoedu/suscribete/

=
Instruções para entrar na lista, sair da lista e usar a lista em
http://www.mat.puc-rio.br/~obmlistas/obm-l.html
=


[obm-l] Re: [obm-l] Socorro em geometria (construçã o)

2012-09-08 Por tôpico Julio César Saldaña



Contrói o triângulo ACT com lados AC=p, CT=q e AT=s.

Por T passe uma paralela a AT. Com centro em A desenhe um arco de raio a. Esse
arco vai cortar à paralela (tem 2 soluções). Chame de D a ese ponto de corte.
Por D traçe uma paralea a CT, o ponto de corte entre essa paralea e AT será o
ponto B do trapecio ABCD procurado.

Julio Saldaña


-- Mensaje original ---
De : obm-l@mat.puc-rio.br
Para : obm-l@mat.puc-rio.br
Fecha : Sat, 8 Sep 2012 03:09:59 +
Asunto : [obm-l] Socorro em  geometria (construçã o)





Construir o trapézio ABCD conhecendo a soma das bases AB +CD = s,as diagonais
AC = p e BD = q e o lado AD = a.Justifique. 		 	   		   


__
Si desea recibir, semanalmente, el Boletín Electrónico de la PUCP, ingrese a:
http://www.pucp.edu.pe/puntoedu/suscribete/

=
Instruções para entrar na lista, sair da lista e usar a lista em
http://www.mat.puc-rio.br/~obmlistas/obm-l.html
=


[obm-l] Re: [obm-l] Desigualdade Triangular

2012-04-25 Por tôpico Julio César Saldaña



Parece que faltou disser que AB=CD=1.

Nesse caso, sejam M, N e P   os pontos meios de BD, BC e AD respectivamente.
Então PM=MN=0.5 e NMP=60, então PN=1. Seja Q o ponto meio de CD, então PQ=AC/2
e QN=BD/2. Aplicando a desigualdade triangular no PQN:

PQ+QN = PN

então

AC/2+BD/2=0.5

AC+BD=1

Julio Saldaña


-- Mensaje original ---
De : obm-l@mat.puc-rio.br
Para : obm-l@mat.puc-rio.br
Fecha : Wed, 25 Apr 2012 04:42:06 +0300
Asunto : [obm-l] Desigualdade Triangular



Sejam AB e CD segmentos de comprimento.Se eles se intersectam em O e

m(AOC)=60º,mostre que AC+BD é maior ou igual a 1.

Desde já obrigado!!
		 	   		   


__
Si desea recibir, semanalmente, el Boletín Electrónico de la PUCP, ingrese a:
http://www.pucp.edu.pe/puntoedu/suscribete/

=
Instruções para entrar na lista, sair da lista e usar a lista em
http://www.mat.puc-rio.br/~obmlistas/obm-l.html
=


[obm-l] RE: [obm-l] Re: [obm-l] Desigualdade Triangular

2012-04-25 Por tôpico Julio César Saldaña



é verdade, PN=0,5

obrigado pela correção

Julio Saldaña


-- Mensaje original ---
De : obm-l@mat.puc-rio.br
Para : obm-l@mat.puc-rio.br
Fecha : Wed, 25 Apr 2012 14:17:16 +
Asunto : RE: [obm-l] Re: [obm-l] Desigualdade Triangular


PN = 0.5,certo?
Interessante a solução!




From: saldana...@pucp.edu.pe
To: obm-l@mat.puc-rio.br
CC: 
Subject: [obm-l] Re: [obm-l] Desigualdade Triangular

Date: Wed, 25 Apr 2012 07:31:13 -0500



Parece que faltou disser que AB=CD=1.

Nesse caso, sejam M, N e P os pontos meios de BD, BC e AD respectivamente.
Então PM=MN=0.5 e NMP=60, então PN=1. Seja Q o ponto meio de CD, então PQ=AC/2
e QN=BD/2. Aplicando a desigualdade triangular no PQN:

PQ+QN = PN

então

AC/2+BD/2=0.5

AC+BD=1

Julio Saldaña


-- Mensaje original ---
De : obm-l@mat.puc-rio.br
Para : obm-l@mat.puc-rio.br
Fecha : Wed, 25 Apr 2012 04:42:06 +0300
Asunto : [obm-l] Desigualdade Triangular


Sejam AB e CD segmentos de comprimento.Se eles se intersectam em O e
m(AOC)=60º,mostre que AC+BD é maior ou igual a 1.
Desde já obrigado!!
 


__
Si desea recibir, semanalmente, el Boletín Electrónico de la PUCP, ingrese a:
http://www.pucp.edu.pe/puntoedu/suscribete/

=
Instruções para entrar na lista, sair da lista e usar a lista em
http://www.mat.puc-rio.br/~obmlistas/obm-l.html
=
		 	   		   


__
Si desea recibir, semanalmente, el Boletín Electrónico de la PUCP, ingrese a:
http://www.pucp.edu.pe/puntoedu/suscribete/

=
Instruções para entrar na lista, sair da lista e usar a lista em
http://www.mat.puc-rio.br/~obmlistas/obm-l.html
=


[obm-l] Re: [obm-l] Não consegui fazer , preciso de ajud a!!

2012-02-22 Por tôpico Julio César Saldaña



Bom, acredito que tenha uma solução com conceitos mais simples, mas a primeira
coisa que me vem a cabeça é:


Prolongue MN e DB até se encontrarem em algum ponto P. Sejam Q e R as
interseções de MN e DB com AC.

Pelo teorema do quadrilátero completo (caso não seja conocido com ese nome
depois pesquiso o nome em português),:D-R-B_P é uma quaterna harmônica, e também
M-Q-N-P é uma quaterna harmônica.

Analissando a primeira quaterna, como AR é bisectriz de BAD então
necessariamente AP é bisectriz do ângulo exterior em A. Então AP é perpendicular
a AR.

Analissando a segunda quaterna, como no haz harmônico formado por AP, AN, AQ e
AM os raios AQ e AP são perpendiculares (demonstrado anteriormente), então
necessariamente cada um de eles é bisectriz de ángulos formados pelos outros dos
raios. Ou seja AQ é bisectriz de NAM, e adicionalmente AP é bisectriz do ângulo
adyacente ao ângulo MAN.

Prometo pensar numa solução com conceitos mais simples.




Julio Saldaña


-- Mensaje original ---
De : obm-l@mat.puc-rio.br
Para : obm-l@mat.puc-rio.br
Fecha : Tue, 21 Feb 2012 17:22:56 -0200
Asunto : [obm-l] Não consegui fazer , preciso de ajud a!!
 

Não consegui fazer , gostaria de uma ajuda!!! Obrigado!! 


Dado um
quadrilatero ABCD , tal que sua diagonal AC seja bissetriz BAD, toma-se
um ponto M no lado CD e traca-se o segmento BM que intercepta AC em F,
em seguida traca-se o segmento DF que intercepta BC em N, mostrar que AC
tambem é bissetriz do angulo MAN! 

Att: Douglas Oliveira 

  


__
Si desea recibir, semanalmente, el Boletín Electrónico de la PUCP, ingrese a:
http://www.pucp.edu.pe/puntoedu/suscribete/

=
Instruções para entrar na lista, sair da lista e usar a lista em
http://www.mat.puc-rio.br/~obmlistas/obm-l.html
=


[obm-l] Re: [obm-l] Re: [obm-l] Não consegui fazer , preciso de ajud a!!

2012-02-22 Por tôpico Julio César Saldaña



O teorema que eu aprendí com o nome do \teorema del cuadrilátero completo\ 
debe
existir nos textos de geometria com outro nome. Vou enunciar, assim tal vez seja
mais facil de identificar.

Dado o quadrilatero ABCD tal que as prolongações de AB e DC se encontram em M,
as prolongações de BC e DA se encontram em B, a prolongação de DB encontra MN em
P e a prolongação de CA encontra à prolongação de MN em Q, então M,P,N e Q
formam uma quaterna harmônica, ou seja:

MP/PN=MQ/QN

Eu sei demonstrar isso usando os teoremas de Ceva e Menelão, mais lembro ter
visto demonstrações mais simples.

No caso do problema que você enviou, apliquei ese teorema ao quadrilátero MCFN.

Com as mesmas condições enunciadas tem outra quaternas harônicas, por exemplo se
T é a interseção de AC e BD então PBTD é uma quaterna harmônica.

Com certeza alguem da lista, com maiores conhecimentos neste tema pode nos
ajudar a achar algum artigo.


Julio Saldaña


-- Mensaje original ---
De : obm-l@mat.puc-rio.br
Para : obm-l@mat.puc-rio.br
Fecha : Wed, 22 Feb 2012 20:51:22 -0200
Asunto : Re: [obm-l] Re: [obm-l] Não consegui fazer  , preciso de ajud a!!
 


Bom , muito obrigado , de qualquer forma vou pesquisar este tal
quadrilátero que não conheco, tem uma ideia de algum artigo??? 


On Wed,
22 Feb 2012 12:05:55 -0500 (PET), Julio César Saldaña wrote: 


Bom,

acredito que tenha uma solução com conceitos mais simples, mas a
primeira

coisa que me vem a cabeça é:

Prolongue MN e DB até se

encontrarem em algum ponto P. Sejam Q e R as

interseções de MN e DB

com AC.


Pelo teorema do quadrilátero completo (caso não seja

conocido com ese nome

depois pesquiso o nome em português),:D-R-B_P é

uma quaterna harmônica, e também

M-Q-N-P é uma quaterna harmônica.




Analissando a primeira quaterna, como AR é bisectriz de BAD então


necessariamente AP é bisectriz do ângulo exterior em A. Então AP é
perpendicular

a AR.

Analissando a segunda quaterna, como no haz

harmônico formado por AP, AN, AQ e

AM os raios AQ e AP são

perpendiculares (demonstrado anteriormente), então

necessariamente

cada um de eles é bisectriz de ángulos formados pelos outros dos



raios. Ou seja AQ é bisectriz de NAM, e adicionalmente AP é bisectriz do
ângulo

adyacente ao ângulo MAN.

Prometo pensar numa solução com

conceitos mais simples.


Julio Saldaña

-- Mensaje original

---

De : obm-l@mat.puc-rio.br [1]
Para : obm-l@mat.puc-rio.br

[2]Fecha : Tue, 21 Feb 2012 17:22:56 -0200

Asunto : [obm-l] Não

consegui fazer , preciso de ajud a!!



Não consegui fazer , gostaria

de uma ajuda!!! Obrigado!! Dado um quadrilatero ABCD , tal que sua
diagonal AC seja bissetriz BAD, toma-se um ponto M no lado CD e traca-se
o segmento BM que intercepta AC em F, em seguida traca-se o segmento DF
que intercepta BC em N, mostrar que AC tambem é bissetriz do angulo MAN!
Att: Douglas Oliveira




__

Si

desea recibir, semanalmente, el Boletín Electrónico de la PUCP, ingrese
a:

http://www.pucp.edu.pe/puntoedu/suscribete/ [3]



=



Instruções para entrar na lista, sair da lista e usar a lista em



http://www.mat.puc-rio.br/~obmlistas/obm-l.html [4]



=




Links:
--
[1] mailto:obm-l@mat.puc-rio.br
[2]
mailto:obm-l@mat.puc-rio.br
[3]
http://www.pucp.edu.pe/puntoedu/suscribete/
[4]
http://www.mat.puc-rio.br/~obmlistas/obm-l.html



__
Si desea recibir, semanalmente, el Boletín Electrónico de la PUCP, ingrese a:
http://www.pucp.edu.pe/puntoedu/suscribete/

=
Instruções para entrar na lista, sair da lista e usar a lista em
http://www.mat.puc-rio.br/~obmlistas/obm-l.html
=


[obm-l] Re: [obm-l] Re: [obm-l] Não consegui fazer , preciso de ajud a!!

2012-02-22 Por tôpico Julio César Saldaña



Achei isto colocando \cuadrilátero completo\ em google:

http://www.aloj.us.es/rbarroso/trianguloscabri/sol/sol168sat.htm

Julio Saldaña


-- Mensaje original ---
De : obm-l@mat.puc-rio.br
Para : obm-l@mat.puc-rio.br
Fecha : Wed, 22 Feb 2012 20:51:22 -0200
Asunto : Re: [obm-l] Re: [obm-l] Não consegui fazer  , preciso de ajud a!!
 


Bom , muito obrigado , de qualquer forma vou pesquisar este tal
quadrilátero que não conheco, tem uma ideia de algum artigo??? 


On Wed,
22 Feb 2012 12:05:55 -0500 (PET), Julio César Saldaña wrote: 


Bom,

acredito que tenha uma solução com conceitos mais simples, mas a
primeira

coisa que me vem a cabeça é:

Prolongue MN e DB até se

encontrarem em algum ponto P. Sejam Q e R as

interseções de MN e DB

com AC.


Pelo teorema do quadrilátero completo (caso não seja

conocido com ese nome

depois pesquiso o nome em português),:D-R-B_P é

uma quaterna harmônica, e também

M-Q-N-P é uma quaterna harmônica.




Analissando a primeira quaterna, como AR é bisectriz de BAD então


necessariamente AP é bisectriz do ângulo exterior em A. Então AP é
perpendicular

a AR.

Analissando a segunda quaterna, como no haz

harmônico formado por AP, AN, AQ e

AM os raios AQ e AP são

perpendiculares (demonstrado anteriormente), então

necessariamente

cada um de eles é bisectriz de ángulos formados pelos outros dos



raios. Ou seja AQ é bisectriz de NAM, e adicionalmente AP é bisectriz do
ângulo

adyacente ao ângulo MAN.

Prometo pensar numa solução com

conceitos mais simples.


Julio Saldaña

-- Mensaje original

---

De : obm-l@mat.puc-rio.br [1]
Para : obm-l@mat.puc-rio.br

[2]Fecha : Tue, 21 Feb 2012 17:22:56 -0200

Asunto : [obm-l] Não

consegui fazer , preciso de ajud a!!



Não consegui fazer , gostaria

de uma ajuda!!! Obrigado!! Dado um quadrilatero ABCD , tal que sua
diagonal AC seja bissetriz BAD, toma-se um ponto M no lado CD e traca-se
o segmento BM que intercepta AC em F, em seguida traca-se o segmento DF
que intercepta BC em N, mostrar que AC tambem é bissetriz do angulo MAN!
Att: Douglas Oliveira




__

Si

desea recibir, semanalmente, el Boletín Electrónico de la PUCP, ingrese
a:

http://www.pucp.edu.pe/puntoedu/suscribete/ [3]



=



Instruções para entrar na lista, sair da lista e usar a lista em



http://www.mat.puc-rio.br/~obmlistas/obm-l.html [4]



=




Links:
--
[1] mailto:obm-l@mat.puc-rio.br
[2]
mailto:obm-l@mat.puc-rio.br
[3]
http://www.pucp.edu.pe/puntoedu/suscribete/
[4]
http://www.mat.puc-rio.br/~obmlistas/obm-l.html



__
Si desea recibir, semanalmente, el Boletín Electrónico de la PUCP, ingrese a:
http://www.pucp.edu.pe/puntoedu/suscribete/

=
Instruções para entrar na lista, sair da lista e usar a lista em
http://www.mat.puc-rio.br/~obmlistas/obm-l.html
=


[obm-l] Re: [obm-l] Re: [obm-l] Dúvida

2011-10-24 Por tôpico Julio César Saldaña



O mesmo visto de outro modo:

Lucas e Pedro tem a mesma velocidade. Então quando lucas sai da ponte, pedro
percorriou 2/5 da ponta, ou seja falta ainda 1/5 da ponte. Então o trem percorre
a ponte inteira no mesmo tempo que pedro percorre 1/5 da ponte, ouseja o trem é
5 vezes mais veloz que pedro, portanto su velocidade é 5x15=75.

Julio Saldaña


-- Mensaje original ---
De : obm-l@mat.puc-rio.br
Para : obm-l@mat.puc-rio.br
Fecha : Mon, 24 Oct 2011 17:08:45 -0200
Asunto : [obm-l] Re: [obm-l] Dúvida

Lucas corre 2/5 da ponte com velocidade de 15 km/h. Sendo p o comprimento da
ponte, leva (2/5)p/15 = 2p/75 h para sair da ponte.
Pedro corre 3/5 da ponte com velocidade de 15 km/h. Sendo p o comprimento da
ponte, leva (3/5)p/15 = 3p/75 h para sair da ponte.

A diferença entre o momento em que Lucas sai da ponte (momento em que o trem
entra na ponte) e o momento em que Pedro sai dela (momento em que o trem sai
da ponte) é 3p/75 - 2p/75 = p/75. Este é o tempo que o trem leva para
percorrer a ponte, ou seja, para percorrer p km.

Portanto, a velocidade do trem será dada por p/(p/75) = 75 km/h, alternativa
C

Abraços.

Hugo.

Em 24 de outubro de 2011 15:29, Jorge Paulino da Silva Filho 
jorge...@yahoo.com.br escreveu:


**
Dois amigos, Lucas e Pedro, seguiam o leito de uma ferrovia e começaram a
atravessar uma ponte estreita na qual havia espaço apenas para o trem. No
momento em que completavam 2/5 do percurso da ponte, ouviram o trem que
se aproxima por trás deles. Lucas começou a correr de encontro ao trem,
saindo da ponte praticamente no instante em que o trem entrava. Pedro,
que correu no sentindo oposto ao sentido de Lucas, conseguir sair da ponte
praticamenteno instante em que o trem saía. Sendo 15km/h a velocidade que
Lucas
e Pedro correram, assinale a alternativa que contém a velocidade do trem:

a)60km/h
b)37km/h
c)75km/h
d)30km/h
e)67,5km/h




__
Si desea recibir, semanalmente, el Boletín Electrónico de la PUCP, ingrese a:
http://www.pucp.edu.pe/puntoedu/suscribete/

=
Instruções para entrar na lista, sair da lista e usar a lista em
http://www.mat.puc-rio.br/~obmlistas/obm-l.html
=


[obm-l] Re: [obm-l] Re: [obm-l] Questão de geometria plana!! ajuda em n ova solução

2011-09-10 Por tôpico Julio César Saldaña



você conhece a solução que usa congruência de triângulos e areas?

Julio Saldaña


-- Mensaje original ---
De : obm-l@mat.puc-rio.br
Para : obm-l@mat.puc-rio.br
Fecha : Sat, 10 Sep 2011 17:17:42 -0300
Asunto : [obm-l] Re: [obm-l] Questão de geometria plana!! ajuda em n ova solução

Ué, deslizar o triângulo pra baixo já é sintético. E é a única que eu
imagino agora.

Em 10/09/11,

douglas.olive...@grupoolimpo.com.brdouglas.olive...@grupoolimpo.com.br

escreveu:



Olá boa tarde, estou com uma questão de geometria plana, que diz
assim: Em um triângulo equilátero, um ponto P interno dista de sues
vértices 5 , 7, e 8 de sues vértices, achar o lado.

gostaria de uma
ajudinha, para elaborar uma nova solução, pois conheco a do oswaldo
dolce, que transporta um triângulo( esta solucao encontra-se no gabarito
do livro de matematica elementar numero 9), tambem conheco uma fazendo
duas leis dos cossenos, e tambem elaborei uma em geometria analitica
fazendo distancia de ponto a ponto , gostaria de uma ajuda para elaborar
outra mas totalmente voltada para geometria plana, dede ja agradeco.




--
/**/
神が祝福

Torres

=
Instruções para entrar na lista, sair da lista e usar a lista em
http://www.mat.puc-rio.br/~obmlistas/obm-l.html
=



__
Si desea recibir, semanalmente, el Boletín Electrónico de la PUCP, ingrese a:
http://www.pucp.edu.pe/puntoedu/suscribete/

=
Instruções para entrar na lista, sair da lista e usar a lista em
http://www.mat.puc-rio.br/~obmlistas/obm-l.html
=


[obm-l] Re: Fwd: [obm-l] Problema de Geometria(difícil)

2011-09-05 Por tôpico Julio César Saldaña



Oi Douglas, resolvi o problema mas não sei se a minha solução é mais simples que
a sua. Acho um pouco complicada, tal vez exista uma solução melhor.


Primeiro vou resumir alguns resultados trivias que você deve ter obtido no 
início:


RDT=TCS=45
RPS=DTC

Para demonstrar que RPST é inscriptivel, basta demonstrar que RPS+RTS=180, ou
seja basta demonstrar que RTS+DTC=180 ou seja, basta demonstrar que 
RTD+CTS=180.


O procedimento vai ser achar outro ángulo na figura igual a RTD e outro igual a
CTS e provar que a soma desses novos 2 ángulos é 180.

Trace uma linha paralela a AD passando por T. Seja M o ponto de interseção dessa
linha com a diagonal BD. Então TMC=45 (=RDT), logo o quadrilátero DRMT é
inscriptível, portanto DMR=RTD. Prono, achamos outro ángulo igual a RTD.

Similarmente, se N é ponto de corte entre aquela linha paralela a AD e a
diagonal AC, então CNS=CTS.

Agora devemos provar que DMR+CNS=180. Este problema é mais fácil. Sugiro fazer
um novo desenho só para este novo problema.

Temos o quadrado ABCD, e uma linha paralela a AD que corta em M à diagonal AC e
em N à diagonal BD. Chamemos de P e Q os pontos de BC e AB que cortam a essa
linha que passa por M e N. Bom, o problema agora é demostrar que AMD e CNB somam
180. Note que: AMD=45+PDM, e CNB=45+PCN então AMD+CNB=90+PDM+PCM. Vou provar que
os triângulos PDM e PNC são congruentes e portanto PDM+PCM=90. Vamos lá:
chamemos PD=a e PC=b, então AQ=a, então QM=a, e como o lado do quadrado é a+b
então PM=b. Similarmente: BQ=b então NQ=b então PN=a, então fica provado que os
triângulos BMP e PCN são congruentes, logo PDM+PCM=90, logo
AMD+CNB=90+PDM+PCM=180, logo RTD+CTS=180 logo RPST é inscriptivel


Me desculpe o portunhol, se algum passo não tiver ficado claro, me avise.

Julio Saldaña


-- Mensaje original ---
De : obm-l@mat.puc-rio.br
Para : obm-l@mat.puc-rio.br
Fecha : Mon, 05 Sep 2011 19:03:50 -0300
Asunto : Fwd: [obm-l] Problema de Geometria(difícil)
 

 Original Message  


SUBJECT:
[obm-l] Problema
de Geometria(difícil)

DATE:
Sun, 04 Sep 2011 11:11:26
-0300

FROM:

douglas.olive...@grupoolimpo.com.br

TO:

REPLY-TO:

obm-l@mat.puc-rio.br

Olá gostaria de uma ajuda no seguinte problema ,
tentei por alguns caminhos, muito trabalhosos, mas que deu certo, não
vou comentar a minha forma de resolução para que tenha criatividade nos
pensamentos, eu estava interessado na forma de resolução por plana!!! ai
vai: 


Dado um quadrado ABCD e um ponto P pertencente ao lado AB , sendo
R o incentro do triângulo APD, S o incentro do triângulo PBC, e T o
incentro do triângulo PCD, mostre que o quadrilátero RPST é inscritível.


E obrigado! 

  


__
Si desea recibir, semanalmente, el Boletín Electrónico de la PUCP, ingrese a:
http://www.pucp.edu.pe/puntoedu/suscribete/

=
Instruções para entrar na lista, sair da lista e usar a lista em
http://www.mat.puc-rio.br/~obmlistas/obm-l.html
=


[obm-l] Re: [obm-l] Geometria OBM

2011-07-24 Por tôpico Julio César Saldaña



Uma solução geométrica:

Sabemos que O2A=O2B.
Prolongue ou estique BO1, (desculpem o protunhol) até um ponto P tal que O2P=O2B
(=O2A). Calculemos uns ángulos: BPO2=20, PO2C=40, AO2P=60, então o triángulo
O2AP é equilátero, ouseja PA=AO2, PAC=30 e a reta AO1 será mediatriz de PO2.
Então PO1=O1O2, então O1O2P=20, finalmente  BO1O2=20+20=40



Julio Saldaña


-- Mensaje original ---
De : obm-l@mat.puc-rio.br
Para : obm-l@mat.puc-rio.br
Fecha : Sun, 24 Jul 2011 19:18:18 -0300
Asunto : Re: [obm-l] Geometria OBM

Olá João ,

Houve um erro na digitação : onde está  AO1 =y  lê-se  BO1 =y . ok ? .
Desculpe o erro .

Abraços

Carlos  Victor

Em 24 de julho de 2011 19:11, Carlos Victor victorcar...@globo.comescreveu:


Olá João ,

Vamos inicialmente a uma solução trigonométrica :

Seja z o ângulo pedido .Sejam também AB =a ; AO2 = x  e AO1= y.Então
teremos :

Triang  *BO1O2 : y/sen(160-z) = x/sen*z

Triang ABO2 : x/sen50 = a/sen80

Triang BO1A : y/sen80 = a/sen70

Logo :sen(20+z) = 4cos10.sen20.senz  ; ou  sen(20+z) -senz = 2senz.sen10

Donde  cos(z+10) = senz ; ou seja  z = 40 ° .

Tentarei uma solução geométrica . ok ?

Abraços

Carlos  Victor






2011/7/24 João Maldonado joao_maldona...@hotmail.com


 Inglaterra -- 1970

No triângulo ABC, AB = AC e A=80°,  dado O1 em AC tal que O1BC =  20° e O2
em  BC tal que   CAO2 = 30°,  calcule   BO1O2

Obrigado
João







__
Si desea recibir, semanalmente, el Boletín Electrónico de la PUCP, ingrese a:
http://www.pucp.edu.pe/puntoedu/suscribete/

=
Instruções para entrar na lista, sair da lista e usar a lista em
http://www.mat.puc-rio.br/~obmlistas/obm-l.html
=


[obm-l] Re: [obm-l] Como demonstrar

2011-07-04 Por tôpico Julio César Saldaña



O que você está calculando é:

(1+1)(1+1/2)(1+1/3)(1+1/n) - 1

= 2.(3/2).(4/3).((n+1)/n) -1

= (n+1) -1

= n

Julio Saldaña


-- Mensaje original ---
De : obm-l@mat.puc-rio.br
Para : obm-l@mat.puc-rio.br
Fecha : Mon, 4 Jul 2011 16:34:12 +
Asunto : [obm-l] Como demonstrar


Resolvendo uma questao me deparei com o seguinte:

1+1/2+1/3+1*1/2*1/3+1*1/2+1*1/3+1/2*1/3 = 3

Considerando 1,1/2,1/3,...1/n,tenho forte impressao de que a soma de todas as

parcelas mais o produto delas mais a soma dos produtos duas a duas mais a soma
dos produtos tres a tres...mais a soma dos produtos n-1 a n-1 é igual a n.

Sendo verdade,como demonstrar?
 		 	   		   


__
Si desea recibir, semanalmente, el Boletín Electrónico de la PUCP, ingrese a:
http://www.pucp.edu.pe/puntoedu/suscribete/

=
Instruções para entrar na lista, sair da lista e usar a lista em
http://www.mat.puc-rio.br/~obmlistas/obm-l.html
=


[obm-l] Re: [obm-l] Geometria

2011-04-26 Por tôpico Julio César Saldaña



Dado que BQC=50=QBC = QC=BC

Seja R um ponto de PB tal que RC=CB. Então BRC=80, RCB=20 = RCQ=60 =
triângulo RQC é equilátero = RQ=QC=RC=BC. De outro lado, como RCP=RPC=40 =
PR=RC, logo PR=RQ, ou seja que o triângulo PRQ é isósceles. Como PRQ=40 (180 -
60 - 80), então RPQ=RQP=70, portanto CPQ=70-40=30.

Julio Saldaña


-- Mensaje original ---
De : obm-l@mat.puc-rio.br
Para : obm-l@mat.puc-rio.br
Fecha : Tue, 26 Apr 2011 20:22:00 -0300
Asunto : [obm-l] Geometria


O seguinte problema está no livro  Geometria I de Morgado, e não sei porque 

não estou conseguindo resolvê-lo. Sei que a resposta é 30º, se alguém  puder
ajudar fico grato.


Em um triângulo isósceles ABC, se base BC, o ângulo  vale 20º. P é um ponto

sobre AB tal que o ângulo PCB = 60º. Q é um ponto em AC tal que QBC = 50º. Qual
a medida do ângulo CPQ?
[]\'sJoão 		 	   		   


__
Si desea recibir, semanalmente, el Boletín Electrónico de la PUCP, ingrese a:
http://www.pucp.edu.pe/puntoedu/suscribete/

=
Instruções para entrar na lista, sair da lista e usar a lista em
http://www.mat.puc-rio.br/~obmlistas/obm-l.html
=


[obm-l] Re: [obm-l] Re: [obm-l] FW: [obm-l] Re: [obm-l] RE: [obm-l] Olim píadas cearenses(geometria)

2011-04-21 Por tôpico Julio César Saldaña



Essa propriedade é valida, não só para trapecios, mas para qualquer cuadilátero.

No cuadrilátero que tem por vertices os pontos medios do primeiro, cada par de
lados opostos são paralelos a uma diagonal do primeiro, e iguai à metade de
esta. Logo, é um paralelogramo.

Julio Saldaña


-- Mensaje original ---
De : obm-l@mat.puc-rio.br
Para : obm-l@mat.puc-rio.br
Fecha : Thu, 21 Apr 2011 18:30:00 -0300
Asunto : [obm-l] Re: [obm-l] FW: [obm-l] Re: [obm-l] RE: [obm-l] Olim píadas
cearenses(geometria)

Tem razão, Marcone e João... relendo agora entendi melhor a questão.
Abs.

Hugo.

Em 21 de abril de 2011 14:28, marcone augusto araújo borges 
marconeborge...@hotmail.com escreveu:




--
From: marconeborge...@hotmail.com
To: obm-l@mat.puc-rio.br
Subject: RE: [obm-l] Re: [obm-l] RE: [obm-l] Olimpíadas
cearenses(geometria)
Date: Thu, 21 Apr 2011 16:05:12 +


 O João está certo.E minha pergunta no final foi se basta provar que DOIS
lados opostos são paralelos e congruentes.Um abraço.
--
Date: Wed, 20 Apr 2011 23:10:44 -0300
Subject: [obm-l] Re: [obm-l] RE: [obm-l] Olimpíadas cearenses(geometria)
From: hfernande...@gmail.com

To: obm-l@mat.puc-rio.br

Não entendi bem sua solução, João.

Pelo que diz o enunciado, os vértices do trapézio são os pontos médios de
um quadrilátero convexo.
Da maneira como você fez, parece que você considerou o quadrilátero formado
pelos pontos médios dos lados do trapézio.
É isso mesmo, ou estou enganado?

Abs.

Hugo.

Em 20 de abril de 2011 20:38, João Maldonado

joao_maldona...@hotmail.comescreveu:


*Na verdade basta provar que os lados opostos são iguais, automaticamente
serão paralelos.*
*Todo uq**adrilátero com ladosopostos iguais é um paralelogramo.*
*
*
*Prova:*
*Faça dois lados (a e b) de um quadrilátero qualquer saindo de um vértice
V. Para que os lados opostos sejm iguais podemos traçar uma circunferência a
partir do fim dos lados a e b, com raio igual ao lado oposto. Desse modo
teríamos 2 circunferências, que se intersectam em 2 pontos. Um dos pontos
gera uma configuração de quadrilátero não convexo, a  outra gera um
quadrilátero convexo. Logo os lados são paralelos.*
*
*
*Mas voltando ao problema,*
*
*
*
*
*Fazendo o trapézio ABCD com lados paralelos AB e CD. Os pontos médios de
AB=X, BC=Y, CD=Z, DA=W. A altura do trapézio h que parte de A intersecta CD
em P e a altura do trapézio que parte de B instersecta CD em Q (neste caso
fazendo P e Q dentro do segmento CD (fica para você provar quando um está
fora). Chamando AB/2 de d, PD de a e QC de b, temos que:*
*1) Em relação a CD, a coordenada y de W é  h/2, e a coordenada x é
(2d+a+b)/2 - a/2 = d+b/2*
*2) Em relação a AB (que é paralela a CD), logo em relação  a CD, a
coordenada y de W é h/2 e a coordenada x é d+b/2, logo os Ângulos formados
com  AB são iguais e as retas WZ e XY são paralelas e de mesma  medida*
*Analogamente para YZ e WX.*
*
*
*Logo se trata de um paralelogramo*
*
*
*
*
*[]\'s*
*João*
*
*
 *
*
 --
From: marconeborge...@hotmail.com
To: obm-l@mat.puc-rio.br
Subject: [obm-l] Olimpíadas cearenses(geometria)
Date: Wed, 20 Apr 2011 22:06:10 +


Prove que um quadrilatero convexo cujos vertices sao os pontos medios dos
lados de um trapezio qualquer é um paralelogramo
Bastaria provar que dos lados opostos são paralelos e congruentes?






__
Si desea recibir, semanalmente, el Boletín Electrónico de la PUCP, ingrese a:
http://www.pucp.edu.pe/puntoedu/suscribete/

=
Instruções para entrar na lista, sair da lista e usar a lista em
http://www.mat.puc-rio.br/~obmlistas/obm-l.html
=


[obm-l] Variáveis Aleatórias

2011-04-10 Por tôpico Julio César Saldaña



Alguem conhece este problema:

Sejam X, Y, Z tres variáveis aleatórias tais que:

E{X.Y} = sigma(X) . sigma(Y)
E{Y.Z} = sigma(Y) . sigma(Z).

Provar que E{X.Z} = sigma(X) . sigma(Z)

Eu achava que o professor tinha se enganado mas já me disse que o porblema é
assi mesmo.

Obrigado
Julio Saldaña



__
Si desea recibir, semanalmente, el Boletín Electrónico de la PUCP, ingrese a:
http://www.pucp.edu.pe/puntoedu/suscribete/

=
Instruções para entrar na lista, sair da lista e usar a lista em
http://www.mat.puc-rio.br/~obmlistas/obm-l.html
=


[obm-l] Re: [obm-l] questao

2011-04-09 Por tôpico Julio César Saldaña



Seja F o ponto de CD tal que a reta AF é perpendicular a reta BE. Ou seja a
prolongação de BE intersecta a AF em M tal que AMB=90.

Então o triângulo AFB é isósceles (AB=BF).

Também FAB=80 e portanto CAF=40, e como ACD também é 40 então:

CF=AF   . (1)

Como BM é mediatriz de AF então EF=EA, mas como EAF=60, então o triângulo AFE é
equilátero, ou seja:

EF=AF   ..(2)

Observe as equações (1) e (2):

em conclusão CF=FE, então FCE=FEC, e como EFD=20 então finalmente FCE=10.

Desculpe o portunhol, mas se algum passo não tiver ficado claro, me avise.

Abraços

Julio Saldaña


-- Mensaje original ---
De : obm-l@mat.puc-rio.br
Para : obm-l@mat.puc-rio.br
Fecha : Sat, 9 Apr 2011 14:27:33 -0700 (PDT)
Asunto : [obm-l] questao

Olá, companheiros!Eis a questão:
Num triângulo ABC, A = 120º, B = 20º e C = 40º. A ceviana AD é tal que BAD mede

20º. A bissetriz interna de B intersecta AD no ponto E. Determine a medida do
ângulo DCE.

Apliquei dois teoremas dos senos, um teorema da bissetriz, prostaférese, arco

duplo e sacanagem até chegar em 10º. Alguém tem uma solução menos
trigonométrica?Um abraço!Grego 


__
Si desea recibir, semanalmente, el Boletín Electrónico de la PUCP, ingrese a:
http://www.pucp.edu.pe/puntoedu/suscribete/

=
Instruções para entrar na lista, sair da lista e usar a lista em
http://www.mat.puc-rio.br/~obmlistas/obm-l.html
=


[obm-l] Re: [obm-l] Termo Geral de uma sequência

2011-04-07 Por tôpico Julio César Saldaña



Oi Emanuel, não sei se tem uma regra geral, vou pesquisar. Neste cado eu daria
estas fórmulas

Para 0 1 2 0 1 2 0 1 2

Se começar com n=0:  a(n) = n - 3[n/3]   ([x] maior inteiro menor o igual a x)
Se começar com n=1:  a(n) = n-1 - 3[(n-1)/3]


Para 0 3 0 3 ...

Se começar com n=0:  a(n) = 1,5*(1 - (-1)^n)
Se começar com n=1:  a(n) = 1,5*(1 + (-1)^n)

Julio Saldaña


-- Mensaje original ---
De : obm-l@mat.puc-rio.br
Para : obm-l@mat.puc-rio.br, emanuelvale...@gmail.com
Fecha : Thu, 7 Apr 2011 00:39:20 -0300
Asunto : [obm-l] Termo Geral de uma sequência

Olá pessoal,

tenho muita dificuldade em calcular o termo geral de sequências do tipo:


0,1,2,0,1,2,...
0,3,0,3,0,3,...

Existe alguma técnica??

abraços,

--
Emanuel

=
Instruções para entrar na lista, sair da lista e usar a lista em
http://www.mat.puc-rio.br/~obmlistas/obm-l.html
=



__
Si desea recibir, semanalmente, el Boletín Electrónico de la PUCP, ingrese a:
http://www.pucp.edu.pe/puntoedu/suscribete/

=
Instruções para entrar na lista, sair da lista e usar a lista em
http://www.mat.puc-rio.br/~obmlistas/obm-l.html
=


[obm-l] Re: [obm-l] QUESTAO DE TRIANGULOS

2011-04-07 Por tôpico Julio César Saldaña



Sobre o lado AC, construa um triângulo equilátero APC, sendo P um ponto externo
ao triângulo ABC (os segmentos BP e AC devem se intersecar num ponto interior a 
AC).

Dado que AP=AB e PAB=60+100=160,  então APB=ABP=10.

Então PBC=40-10=30

Os triângulos PCB e CAD são congruentes (iguais, caso LAL).

então ADC=PBC=30, então BCD=40-30=10 




Julio Saldaña


-- Mensaje original ---
De : obm-l@mat.puc-rio.br
Para : obm-l@mat.puc-rio.br
Fecha : Wed, 6 Apr 2011 09:10:29 -0700 (PDT)
Asunto : [obm-l] QUESTAO DE TRIANGULOS
Num triângulo ABC tem-se que o ângulo ABC  é igual ao ângulo ACB que vale 40 
graus .
Prolongando-se o lado AB, no sentido de A para B, até um ponto D tal que AD 
igual a BC, a medida do

ângulo BCD é igual a:

Felipe Araujo Costa


=
Instruções para entrar na lista, sair da lista e usar a lista em
http://www.mat.puc-rio.br/~obmlistas/obm-l.html
=



__
Si desea recibir, semanalmente, el Boletín Electrónico de la PUCP, ingrese a:
http://www.pucp.edu.pe/puntoedu/suscribete/

=
Instruções para entrar na lista, sair da lista e usar a lista em
http://www.mat.puc-rio.br/~obmlistas/obm-l.html
=


[obm-l] conj untos, difícil

2011-04-06 Por tôpico Julio César Saldaña



Oi Bernardo, muito obrigado pela explicação. Já comecei a gostar deste tema.

Vou continuar estudando este tema que está muito interessante.

Só vou parar alguns dias, pois tenho provas a semana próxima. Após as provas,
volto no assunto.

Obrigado

abraços

Julio Saldaña


-- Mensaje original ---
De : obm-l@mat.puc-rio.br
Para : obm-l@mat.puc-rio.br
Fecha : Mon, 4 Apr 2011 18:17:36 +0200
Asunto : [obm-l] Re: [obm-l] Re: [obm-l] Re: [obm-l] Re: [obm-l] conj untos, 
difícil

2011/4/4 Julio César Saldaña saldana...@pucp.edu.pe:

Oi Samuel e Bernardo, desculpem , acho que eu tinha entendido mal o conceito de
distância.

Oi Julio,


Só para conferir

Se tenho dos círculos de radio 1, e os centros etão ém (0,0) e (0,3), então a
distância entre eles seria: 5, isso é correto?

Acho que eu tinha interpretado errado e achava que distância nesse exemplo é 1.


Vamos lá, com calma. A definição, pra começar:

h(A,B) = inf { r , para cada x em A, existe y em B tq d(x,y)  r e
para cada y em B, existe x em A tq d(x,y)  r}

Ou seja, para todo ponto a de A, você tem um ponto b_a (ou seja, que
pode mudar em função de a) em B a distância menor ou igual a h(A,B) (o
problema do inf é que muda os quantificadores), e o mesmo com b em B,
e um ponto a_b em A. (Aqui, você usa que os conjuntos são compactos
para garantir a desigualdade no limite). O melhor é separar a
definição em \distância de A até B\ e \distância de B até A\, cada uma
sendo uma das metades, e ver que como a gente exige que o \r\ valha
para os dois, então temos que h(A,B) é o máximo dessas duas distâncias
(que não são simétricas, por isso que a gente não as usa)

Se você tem o seu círculo em (0,0) de raio 1 (ele é o meu A), o ponto
\mais longe\ do outro círculo (o B) é (-1,0), mas para todo r  3
existe um ponto (o (2,0) para ser mais exato) que está a uma distância
menor do que r de (-1,0). Os outros pontos (a,b) têm pontos (a+3,b)
correspondentes no outro círculo também, de forma que a \distância de
A até B\ é 3. Como a figura é simétrica, a distância (tal como
definida pelo Samuel) é 3.

Para dar um exemplo um pouco mais interessante, veja que se A =
segmento [0,1] e B = segmento [2,20], a \distância de A até B\ como eu
defini é 2 porque todo ponto em A está a uma distância = a 2 do
[2,20]. Por outro lado, a \distância de B até A\ é 19, porque o ponto
20 está a uma distância de 19 do 1, que é o ponto mais próximo do A.

Agora, de volta ao problema:

Uma forma interessante de ver essa definição da h(A,B) é a seguinte:
defina a \distância entre x e A\ (um conjunto) como a menor distância
entre x e um ponto de A, ou seja, d(x, A) = inf{ d(x,a) / a pertence a
A}. Note que essa distância é sempre realizada quando A é um conjunto
fechado, porque você pode pegar uma seqüência decrescente de
distâncias, e os pontos que as realizam formam uma seqüência em A,
logo qualquer limite está em A, e porque elas estão a uma distância 
constante, isso dá um compacto, donde você pode extrair uma
subseqüência.
Agora, defina d(A - B) = sup{ d(a,B) / a pertence a A} = \distância
de A até B\, e h(A,B) = max{d(A - B), d(B - A)}. Isso quer dizer que
B inter {vizinhança de espessura r  h(A,B) em volta de A} é não vazio
para todo r, e reciprocamente em A e B. Repare que não é o mesmo que
pedir que B esteja contido na \bola em volta de A\ de raio r. (\bola
em volta de A\ = vizinhança de espessura r = conjunto dos pontos cuja
distância a A é menor do que r).

Agora, para concluir o problema, vai uma dica: use a desigualdade
triangular original, mais o fato que h(A,B)  r te dá um ponto em B
para cada ponto de A, e o mesmo para h(B,C)  s para fazer pontos em
C. E depois dê uma jogada de epsilon/2 e pode partir pro abraço.

Abraços,
--
Bernardo Freitas Paulo da Costa

=
Instruções para entrar na lista, sair da lista e usar a lista em
http://www.mat.puc-rio.br/~obmlistas/obm-l.html
=



__
Si desea recibir, semanalmente, el Boletín Electrónico de la PUCP, ingrese a:
http://www.pucp.edu.pe/puntoedu/suscribete/

=
Instruções para entrar na lista, sair da lista e usar a lista em
http://www.mat.puc-rio.br/~obmlistas/obm-l.html
=


[obm-l] Re: [obm-l] QUETAO GEOMETRIA PLANA TRIANGULOS

2011-04-04 Por tôpico Julio César Saldaña



Seja BH a altura relativa a AC, então, como o triângulo é isósceles BH deve ser
mediatriz de AC, e os ángulos ABH e HBC medem 40 cada.

Seja Q o ponto de interseção de BH e CP. Dado que BH é mediatriz de BC, então
AQ=QC e o ângulo QAC mede igual que o ACQ, ou seja 30. Então o ángulo PAC mede
40-30 = 10, e o ángulo PQC mede 60 e o ángulo PQB também 60.

Ou seja, no triângulo ABQ, AP e QP são bisectrices, então P é o incentro do
triângulo ABQ, então BP deve ser bisectriz, por tanto ABP=PBQ=20 e finalmente
BPC=100.

Parece complicado, mas é pelo fato de explicar sem mostrar a figura. Se precissa
de uma figura me avise para enviar como adjunto



Julio Saldaña


-- Mensaje original ---
De : obm-l@mat.puc-rio.br
Para : obm-l@mat.puc-rio.br
Fecha : Sun, 3 Apr 2011 23:22:22 -0700 (PDT)
Asunto : [obm-l] QUETAO GEOMETRIA PLANA TRIANGULOS
Olá, 
Queria uma outra solução mais simples que pela trigonometria. Se alguem 
conseguir???


===
Seja P um ponto do interior de um triângulo isósceles ABC tal que AB igual a 
BC, o angulo ABC VALE 80o ,

o angulo PAC = 40o e o angulo ACP = 30o . A medida do ângulo BPC é igual a:


Obrigado;
Felipe Araujo Costa

=
Instruções para entrar na lista, sair da lista e usar a lista em
http://www.mat.puc-rio.br/~obmlistas/obm-l.html
=



__
Si desea recibir, semanalmente, el Boletín Electrónico de la PUCP, ingrese a:
http://www.pucp.edu.pe/puntoedu/suscribete/

=
Instruções para entrar na lista, sair da lista e usar a lista em
http://www.mat.puc-rio.br/~obmlistas/obm-l.html
=


[obm-l] Re: [obm-l] Re: [obm-l] Re: [obm-l] conjuntos, difícil

2011-04-04 Por tôpico Julio César Saldaña



Oi Samuel e Bernardo, desculpem , acho que eu tinha entendido mal o conceito de
distância.

Só para conferir

Se tenho dos círculos de radio 1, e os centros etão ém (0,0) e (0,3), então a
distância entre eles seria: 5, isso é correto?

Acho que eu tinha interpretado errado e achava que distância nesse exemplo é 1.

Agradeço sua explicação

Abraços

Julio Saldaña


-- Mensaje original ---
De : obm-l@mat.puc-rio.br
Para : obm-l@mat.puc-rio.br
Fecha : Mon, 4 Apr 2011 14:28:00 +0200
Asunto : [obm-l] Re: [obm-l] Re: [obm-l] conjuntos, difícil

2011/4/2 Julio César Saldaña saldana...@pucp.edu.pe:

Oi Samuel,

Na verdade não entendo muito de este tema, e queria te perguntar se os círculos
no plano são subconjuntos compactos do plano?

Sim, círculos são subconjuntos compactos do plano.


Se for assim, se me ocurre um exemplo onde não é verdade:
h(A,C) = h(A,B) + h(B,C)

suponha que tem tres círculos com os centros colineales (na mesma reta). Nesse
caso a distancia entre os mais afastados é maior que a soma das distancias entre
eles e o terceiro:  h(A,C)  h(A,B) + h(B,C)

Isso está certo ou o exemplo não tem nada a ver com o que se quer demostrar?

Eu não entendi o seu exemplo... Você pode dar os centros dos círculos
(três pontos numa reta) e os respectivos raios? E ajudaria a mim e o
Samuel se você calculasse explicitamente quanto valem cada um dos
h(A,B), h(B,C) e h(A,C).

Abraços,
--
Bernardo Freitas Paulo da Costa

=
Instruções para entrar na lista, sair da lista e usar a lista em
http://www.mat.puc-rio.br/~obmlistas/obm-l.html
=



__
Si desea recibir, semanalmente, el Boletín Electrónico de la PUCP, ingrese a:
http://www.pucp.edu.pe/puntoedu/suscribete/

=
Instruções para entrar na lista, sair da lista e usar a lista em
http://www.mat.puc-rio.br/~obmlistas/obm-l.html
=


[obm-l] Re: [obm-l] conjuntos, difícil

2011-04-02 Por tôpico Julio César Saldaña



Oi Samuel,

Na verdade não entendo muito de este tema, e queria te perguntar se os círculos
no plano são subconjuntos compactos do plano?

Se for assim, se me ocurre um exemplo onde não é verdade:
h(A,C) = h(A,B) + h(B,C)

suponha que tem tres círculos com os centros colineales (na mesma reta). Nesse
caso a distancia entre os mais afastados é maior que a soma das distancias entre
eles e o terceiro:  h(A,C)  h(A,B) + h(B,C)

Isso está certo ou o exemplo não tem nada a ver com o que se quer demostrar?


Obrigado


Julio Saldaña


-- Mensaje original ---
De : obm-l@mat.puc-rio.br
Para : obm-l@mat.puc-rio.br
Fecha : Sat, 2 Apr 2011 00:58:01 +
Asunto : [obm-l] conjuntos,  difícil


Seja (M,d) um espaço métrico. Denote por K(M) ao conj. de todos os subconj.

Compactos de M e defina a distância por:


h(A,B) = inf { r , para cada x em A, existe y em B tq d(x,y)  r e para cada y

em B, existe x em A tq d(x,y)  r}


Provar que (K(M) é espaço métrico).

i) h(A,B) = h(B,A)  (consegui fazer).

ii) h(A,B)  0 se A  B  e h(A,A) = 0 (aqui usei o fato de de os conj serem

compactos em um espaço métrico, então Hausdorff, portanto esses conj são fech.)
esse item também consegui fazer.


Agora vem o problemático (para mim) 


iii) h(A,C) = h(A,B) + h(B,C)  para todos A,B,C  pertencenta à K(M).


Queria pedir um socorro nesta última afirmação, não estou conseguindo fazer.

Obrigado. 		 	   		   


__
Si desea recibir, semanalmente, el Boletín Electrónico de la PUCP, ingrese a:
http://www.pucp.edu.pe/puntoedu/suscribete/

=
Instruções para entrar na lista, sair da lista e usar a lista em
http://www.mat.puc-rio.br/~obmlistas/obm-l.html
=


[obm-l] RE: [obm-l] Re: [obm-l] Problema de futebol

2011-04-01 Por tôpico Julio César Saldaña



Esses 5 times, jogam (5x4/2 = 10) jogos, e o máximo número de pontos a ganhar é
10x3 = 30 no total.

O quinto colocado consegue o maior número de pontos quando os 5 times obtém o
mesmo número de pontos 30/5 = 6. Porque se algum consegue mais de 6 pontos,
então algum vai conseguir menos de 6, ou seja com certeza o quinto colocado
teria menos de 6. Por isso o máximo número de pontos do quinto colocado é 6.

Se o quinto colocado consegue mais de 6, então já no é quinto (hehe).

Será que me falta rigor na demonstração?

Obrigado

Julio Saldaña


-- Mensaje original ---
De : obm-l@mat.puc-rio.br
Para : obm-l@mat.puc-rio.br
Fecha : Fri, 1 Apr 2011 10:45:38 +
Asunto : RE: [obm-l] Re: [obm-l] Problema de futebol


Caro Júlio César Saldaña,

Muito obrigado pela resolução. Tenho ainda uma dúvida. É quanto ao trecho 
abaixo:

\ ... o máximo número de pontos que pode ter ganho o quinto colocado é (no caso

que todos os 5

ganharam o mesmo número de pontos) 30/5 =6.\


Um abraço do Paulo Argolo.



From: saldana...@pucp.edu.pe
To: obm-l@mat.puc-rio.br
CC:
Subject: [obm-l] Re: [obm-l] Problema de futebol
Date: Thu, 31 Mar 2011 09:31:00 -0500



Favor analisar esta solução:


Para saber qual é o mínimo número de pontos necessário para ficar nos quatro
primeiros, investiguemos qual é o máximo número de pontos que pode ter o quinto
colocado.

Para chegar nessa situação, suponhamos que os 5 primeiros colocados ganharam
todos os jogos contra os outros 7. Então tem pelo menos 21 pontos cada.

Faltam distribuir os pontos disputados no 10 jogos que esse 5 jogaram entre
eles. O máximo número de pontos a ser distribuídos é 3x10 = 30, assim, o máximo
número de pontos que pode ter ganho o quinto colocado é (no caso que todos os 5
ganharam o mesmo número de pontos) 30/5 =6. Isto corresponde a cada um ter ganho
2 jogos dos 10 disputados, o qual é perfeitamente possível.

Nesse caso os cinco primeiros colocados tem 21+6=27. Ou seja ter 27 pontos, não
garante ficar nos 4 primeiros (pode ficar quinto), e além de mais o máximo
número de pontos do quinto colocado é 27. Portanto o mínimo número de pontos que
garantem ficar nos 4 primeiros é 28.

Rpta: 28

Me avisem se tem algo errado o falta justificar algum passo com maior rigor.

Obrigado




Julio Saldaña


-- Mensaje original ---
De : obm-l@mat.puc-rio.br
Para : obm-l@mat.puc-rio.br
Fecha : Wed, 30 Mar 2011 22:17:44 +
Asunto : [obm-l] Problema de futebol

Caríssimos colegas,


Gostaria de obter, se possível for, uma resolução da questão abaixo:

QUESTÃO:

Um torneio de futebol é disputado por 12 times.Na primeira fase,cada time
enfrenta os demais uma única vez e obtém 1 ponto quando empata e 3 pontos quando
vence. A segunda fase do torneio será disputada somente pelos 4 times que
obtiverem mais pontos na primeira fase — havendo necessidade, será adotado algum
critério de desempate (saldo de gols, por exemplo) para definir quais serão
esses 4 times.
Qual é o número mínimo de pontos que garante a um time sua classificação para a
segunda fase, independentemente do critério de desempate que seja adotado?

Abraços!
Paulo Argolo
=
Instruções para entrar na lista, sair da lista e usar a lista em
http://www.mat.puc-rio.br/~obmlistas/obm-l.html
=


__
Si desea recibir, semanalmente, el Boletín Electrónico de la PUCP, ingrese a:
http://www.pucp.edu.pe/puntoedu/suscribete/

=
Instruções para entrar na lista, sair da lista e usar a lista em
http://www.mat.puc-rio.br/~obmlistas/obm-l.html


=   

 

=
Instruções para entrar na lista, sair da lista e usar a lista em
http://www.mat.puc-rio.br/~obmlistas/obm-l.html
=



__
Si desea recibir, semanalmente, el Boletín Electrónico de la PUCP, ingrese a:
http://www.pucp.edu.pe/puntoedu/suscribete/

=
Instruções para entrar na lista, sair da lista e usar a lista em
http://www.mat.puc-rio.br/~obmlistas/obm-l.html
=


[obm-l] Re: [obm-l] RE: [obm-l] Re: [obm-l] Problema de futebol

2011-04-01 Por tôpico Julio César Saldaña



Muito obrigado Ralph, estou tomando nota desses dois pontos. Acho que assim fica
melhor justificada a resolução

Abraços

Julio Saldaña


-- Mensaje original ---
De : obm-l@mat.puc-rio.br
Para : obm-l@mat.puc-rio.br
Fecha : Fri, 1 Apr 2011 13:58:50 -0300
Asunto : Re: [obm-l] RE: [obm-l] Re: [obm-l] Problema de futebol

Para mim, o raciocinio do Julio parece correto.


Se o quinto colocado consegue mais de 6, então já não é quinto (hehe).


Sim, você usou uma versão do princípio da casa dos pombos. Acho que fica
mais fácil de explicar por contradição: se o 5o tivesse mais que 6, os 5
primeiros teriam mais que 6 cada um, então eles teriam mais de 30 pontos
juntos entre si, absurdo.
De fato, o pedaço que me fez piscar foi outro:


\Isto corresponde a cada um ter ganho 2 jogos dos 10 disputados, o qual

é*perfeitamente possível

*.\

Hmmm Ah, sim: por assim dizer, ponha os 5 times sentados numa mesa
circular, e faça cada um ganhar dos dois times imediatamente à sua direita.
Perfeitamente possível. :)

(A menos que um deles tenha vendido os direitos de TV pelo clube dos 5, mas
o outros tenha feito diretamente uma negociação octa-lateral com os outros
7... ah, problema errado.)

Abraço,
  Ralph

2011/4/1 Julio César Saldaña saldana...@pucp.edu.pe:



Esses 5 times, jogam (5x4/2 = 10) jogos, e o máximo número de pontos a

ganhar é

10x3 = 30 no total.

O quinto colocado consegue o maior número de pontos quando os 5 times

obtém o

mesmo número de pontos 30/5 = 6. Porque se algum consegue mais de 6

pontos,

então algum vai conseguir menos de 6, ou seja com certeza o quinto

colocado

teria menos de 6. Por isso o máximo número de pontos do quinto colocado é

6.


Se o quinto colocado consegue mais de 6, então já no é quinto (hehe).

Será que me falta rigor na demonstração?

Obrigado

Julio Saldaña


-- Mensaje original ---
De : obm-l@mat.puc-rio.br
Para : obm-l@mat.puc-rio.br
Fecha : Fri, 1 Apr 2011 10:45:38 +
Asunto : RE: [obm-l] Re: [obm-l] Problema de futebol


Caro Júlio César Saldaña,

Muito obrigado pela resolução. Tenho ainda uma dúvida. É quanto ao trecho

abaixo:


\ ... o máximo número de pontos que pode ter ganho o quinto colocado é

(no caso

que todos os 5

ganharam o mesmo número de pontos) 30/5 =6.\


Um abraço do Paulo Argolo.



From: saldana...@pucp.edu.pe
To: obm-l@mat.puc-rio.br
CC:
Subject: [obm-l] Re: [obm-l] Problema de futebol
Date: Thu, 31 Mar 2011 09:31:00 -0500



Favor analisar esta solução:


Para saber qual é o mínimo número de pontos necessário para ficar nos

quatro

primeiros, investiguemos qual é o máximo número de pontos que pode ter o

quinto

colocado.

Para chegar nessa situação, suponhamos que os 5 primeiros colocados

ganharam

todos os jogos contra os outros 7. Então tem pelo menos 21 pontos cada.

Faltam distribuir os pontos disputados no 10 jogos que esse 5 jogaram

entre

eles. O máximo número de pontos a ser distribuídos é 3x10 = 30, assim, o

máximo

número de pontos que pode ter ganho o quinto colocado é (no caso que

todos os 5

ganharam o mesmo número de pontos) 30/5 =6. Isto corresponde a cada um

ter ganho

2 jogos dos 10 disputados, o qual é perfeitamente possível.

Nesse caso os cinco primeiros colocados tem 21+6=27. Ou seja ter 27

pontos, não

garante ficar nos 4 primeiros (pode ficar quinto), e além de mais o

máximo

número de pontos do quinto colocado é 27. Portanto o mínimo número de

pontos que

garantem ficar nos 4 primeiros é 28.

Rpta: 28

Me avisem se tem algo errado o falta justificar algum passo com maior

rigor.


Obrigado




Julio Saldaña


-- Mensaje original ---
De : obm-l@mat.puc-rio.br
Para : obm-l@mat.puc-rio.br
Fecha : Wed, 30 Mar 2011 22:17:44 +
Asunto : [obm-l] Problema de futebol

Caríssimos colegas,


Gostaria de obter, se possível for, uma resolução da questão abaixo:

QUESTÃO:

Um torneio de futebol é disputado por 12 times.Na primeira fase,cada

time

enfrenta os demais uma única vez e obtém 1 ponto quando empata e 3

pontos quando

vence. A segunda fase do torneio será disputada somente pelos 4 times

que

obtiverem mais pontos na primeira fase — havendo necessidade, será

adotado algum

critério de desempate (saldo de gols, por exemplo) para definir quais

serão

esses 4 times.
Qual é o número mínimo de pontos que garante a um time sua

classificação para a

segunda fase, independentemente do critério de desempate que seja

adotado?


Abraços!
Paulo Argolo


=

Instruções para entrar na lista, sair da lista e usar a lista em
http://www.mat.puc-rio.br/~obmlistas/obm-l.html


=



__
Si desea recibir, semanalmente, el Boletín Electrónico de la PUCP,

ingrese a:

http://www.pucp.edu.pe/puntoedu/suscribete

[obm-l] Re: [obm-l] Problema de futebol

2011-03-31 Por tôpico Julio César Saldaña



Favor analisar esta solução:

Para saber qual é o mínimo número de pontos necessário para ficar nos quatro
primeiros, investiguemos qual é o máximo número de pontos que pode ter o quinto
colocado.

Para chegar nessa situação, suponhamos que os 5 primeiros colocados ganharam
todos os jogos contra os outros 7. Então tem pelo menos 21 pontos cada.

Faltam distribuir os pontos disputados no 10 jogos que esse 5 jogaram entre
eles. O máximo número de pontos a ser distribuídos é 3x10 = 30, assim, o máximo
número de pontos que pode ter ganho o quinto colocado é (no caso que todos os 5
ganharam o mesmo número de pontos) 30/5 =6. Isto corresponde a cada um ter ganho
2 jogos dos 10 disputados, o qual é perfeitamente possível.

Nesse caso os cinco primeiros colocados tem 21+6=27. Ou seja ter 27 pontos, não
garante ficar nos 4 primeiros (pode ficar quinto), e além de mais o máximo
número de pontos do quinto colocado é 27. Portanto o mínimo número de pontos que
garantem ficar nos 4 primeiros é 28.

Rpta: 28

Me avisem se tem algo errado o falta justificar algum passo com maior rigor.

Obrigado




Julio Saldaña


-- Mensaje original ---
De : obm-l@mat.puc-rio.br
Para : obm-l@mat.puc-rio.br
Fecha : Wed, 30 Mar 2011 22:17:44 +
Asunto : [obm-l] Problema de futebol


Caríssimos colegas,


Gostaria de obter, se possível for, uma resolução da questão abaixo:

QUESTÃO:

Um torneio de futebol é disputado por 12 times.Na primeira fase,cada time

enfrenta os demais uma única vez e obtém 1 ponto quando empata e 3 pontos quando
vence. A segunda fase do torneio será disputada somente pelos 4 times que
obtiverem mais pontos na primeira fase — havendo necessidade, será adotado algum
critério de desempate (saldo de gols, por exemplo) para definir quais serão
esses 4 times.

Qual é o número mínimo de pontos que garante a um time sua classificação para a

segunda fase, independentemente do critério de desempate que seja adotado?


Abraços!
Paulo Argolo 		 	   		  
=

Instruções para entrar na lista, sair da lista e usar a lista em
http://www.mat.puc-rio.br/~obmlistas/obm-l.html
=



__
Si desea recibir, semanalmente, el Boletín Electrónico de la PUCP, ingrese a:
http://www.pucp.edu.pe/puntoedu/suscribete/

=
Instruções para entrar na lista, sair da lista e usar a lista em
http://www.mat.puc-rio.br/~obmlistas/obm-l.html
=


[obm-l] Re: [obm-l] Urna Probabilidade

2011-03-31 Por tôpico Julio César Saldaña



Não sei se é a solução mais elegante, mas..

O evento desejado pode ser representado como a união dos seguintes eventos
disjuntos:

A = A primeira bola foi branca
B = As duas primeiras foram pretas e a terceira foi branca
C = As quatro primeiras foram pretas e a quinta foi branca.

Então a probabilidade pedida é P(A)+P(B)+P(C)

P(A) = 2/6 = 1/3
P(B) = (4/6)(3/5)(2/4)=1/5
P(C) = (4/6)(3/5)(2/4)(1/3)(1) = 1/15


Somando (1/3) + (1/5) + (1/15) = 3/5

Me avisem se fiz errado

Obrigado

Julio Saldaña


-- Mensaje original ---
De : obm-l@mat.puc-rio.br
Para : obm-l@mat.puc-rio.br
Fecha : Wed, 30 Mar 2011 21:10:53 +0300
Asunto : [obm-l] Urna Probabilidade


Prezados.
Em uma urna, são colocadas 2 bolas brancas e 4 pretas.Alberto e Beatriz retiram

bolas da urna alternadamente, iniciando-se com Alberto, até que a urna esteja
vazia. A probabilidade de que a primeira bola branca saia para Alberto é

(A) 1/2
(B) 3/5
(C) 5/9
(D) 7/12
(E) 8/15
Grato.
Marcos. 		 	   		   


__
Si desea recibir, semanalmente, el Boletín Electrónico de la PUCP, ingrese a:
http://www.pucp.edu.pe/puntoedu/suscribete/

=
Instruções para entrar na lista, sair da lista e usar a lista em
http://www.mat.puc-rio.br/~obmlistas/obm-l.html
=


[obm-l] Re: [obm-l] Área do tr iângulo

2011-03-31 Por tôpico Julio César Saldaña



Sejam, a, b e c os lados do triángulo, então o quadrado da area pode ser
expressa assim:

Quadrado da area = p(p-a)(p-b)(p-c), onde p é o semi-perímetro =(a+b+c)/2

ou seja que aquele produto deve ser o quadrado de um número inteiro positivo.

seja S=a+b+c o perímetro, então o quadrado da área é

Quadrado da area = (S/2)(S/2 -a)(S/2 - b)(S/2 -c)

então:

16 . quadrado da area = S(S-2a)(S-2b)(S-2c)


Se S fosse ímpar, então o lado direito da igualdade seria ímpar, o qual é
incorreto pois do lado esquerdo temos um par.

então S deve ser par, ou seaj p=S/2 é um inteiro.

Voltando ao quadrado da area:

Quadrado da area = p(p-a)(p-b)(p-c)

O primeiro fator (p) é a soma dos otros tres p = (p-a) + (p-b) + (p-c)

então temos o problema de encontrar tres número inteiros que multiplicados por a
soma deles o resultado é o quadrado de outro inteiro. Os mínimos números
inteiros que cumprem isso são 1, 2 e 3, pois a soma é 6 e 1.2.3.6 = 36 (quadrado
de um inteiro).

então 


p-a=1
p-b=2
p-c=3

Resolvendo a=5, b=4, c=3







Julio Saldaña


-- Mensaje original ---
De : obm-l@mat.puc-rio.br
Para : obm-l@mat.puc-rio.br
Fecha : Thu, 31 Mar 2011 19:34:01 +0300
Asunto : [obm-l] Área do tr iângulo


   Um triângulo tem que seus lados e sua área são números inteiros
positivos.Qual é o menor valor para a área? 		 	   		   


__
Si desea recibir, semanalmente, el Boletín Electrónico de la PUCP, ingrese a:
http://www.pucp.edu.pe/puntoedu/suscribete/

=
Instruções para entrar na lista, sair da lista e usar a lista em
http://www.mat.puc-rio.br/~obmlistas/obm-l.html
=


[obm-l] Re: [obm-l] fUNÇÃO

2011-03-30 Por tôpico Julio César Saldaña



Para que (fofof)(x) seja 3, e necessário que (fof)(x) seja 2, e para isso é
necessário que f(x) seja 1,. Ou seja x pode ser 0 ou 3. Soma dos valores 3+0 = 
3.



Julio Saldaña


-- Mensaje original ---
De : obm-l@mat.puc-rio.br
Para : obm-l@mat.puc-rio.br
Fecha : Wed, 30 Mar 2011 10:09:17 -0300
Asunto : [obm-l] fUNÇÃO

Alguém me ajuda nessa .

. Considere o conjunto A = {0, 1, 2, 3} e a função f: A #8594; A tal que
f(3) = 1

e f(x) = x + 1, se x #8800; 3 soma dos valores de x para os quais (fofof)(x) = 
3 é:







__
Si desea recibir, semanalmente, el Boletín Electrónico de la PUCP, ingrese a:
http://www.pucp.edu.pe/puntoedu/suscribete/

=
Instruções para entrar na lista, sair da lista e usar a lista em
http://www.mat.puc-rio.br/~obmlistas/obm-l.html
=